You are on page 1of 44

Thyroglossal Cyst

Ectopic Thyroid
- Thyroid located in an abnormal position.
- Lingual thyroid is common, located in the posterior 1/3rd of the tongue.
- Can occur anywhere along the thyroglossal duct.
- Should be differentiated from a thyroglossal cyst to prevent inadvertent removal of the only thyroid gland.
Apthous ulcers
- Canker sores
- 40% of US population
- Unknown etiology
- Painful & recurrent
- Familial predilection
- Shallow, hyperemic ulcers infiltrated by mononuclear
- inflammatory cells & later by neutrophils
Glossitis
- Tongue inflammation
- Beefy-red tongues due to papillae atrophy
- with mucosal thinning
- Sprue, Vit B12, Riboflavin, Niacin, Iron and Pyridoxine deficiencies
Oral candidiasis
- Superficial gray-white inflammatory membranes having fungus in a fibrinosuppurative exudate
- DM, neutropenia and
Tumors and Precancerous Lesions
- Leukoplakia
o Clinical term for a white plaque on oral mucous membranes that cannot be removed by scraping
o Chronic tobacco use
- Benign to Squamous cell carcinoma of the mouth spectrum
- Erythroplakia
o Red, velvety, flat lesion
o Less common than leukoplakia
o Greater risk of malignancy transformation
Hairy leukoplakia
- In HIV patients caused by EBV
- White patches of fluffy (hairy) hyperkeratosis on tongue’s lateral borders
- Superimposed Candida
Oral Squamous Cell Carcinoma
- 95% of oral cancers
- 50-70 yrs
- Mouth floor, tongue, soft palate, lips & tongue base
- Raised, firm, ulcerated or verrucous
- Metastasis-Cervical LN, lungs, liver & bones
- Lip lesions-Best prognosis
- Mouth floor & tongue base lesions-poorest prognosis
- Pathogenesis
o Tobacco & Alcohol
o HPV-6, 16, 18
o Betel nut & paan
o Chromosomes 18q, 10p, 8p & 3p deletions
- A 60-year-old man presents with a 5-week history of difficulty swallowing. Physical examination is unremarkable. Upper
endoscopy shows a large mass in the upper third of the esophagus. A biopsy is shown in the image. What is the appropriate
histologic diagnosis for this esophageal mass? Squamous Cell Carcinoma
Odontogenic Tumors
- True neoplasms or hamartomas
- Origin-Odontogenic epithelium or ectomesenchyme
- Ameloblastoma- True neoplasm arising from odontogenic epithelium. Cystic, slow growing & locally invasive but typically is
benign
- Grossly, the lesions consist of multiple cysts filled with a thick, “motor oil”–like fluid.
- Tumor consisting of nests of tumor cells that appear dark and crowded at the periphery of the nests and loose in the center
(similar to the stellate reticulum of a developing tooth).
Nasopharyngeal carcinoma
- EBV infection
- African children & Southern China adults
- Large epithelial cells with indistinct borders and prominent nuclei
- Nonkeratinizing SCC or undifferentiating carcinoma with abundant lymphocytic infiltrate (lymphoepithelioma)
- Often unresectable with nodal metastasis Most are sensitive to radiotherapy
- Figure: Large epithelial cells with indistinct borders and prominent nuclei and lymphocytic infiltrate.
Vocal cord polyp
- Heavy smokers / singers / politicians
- Small, smooth, rounded excrescences on true vocal cords
- Myxoid, vascular connective tissue covered by squamous epithelium
- Progressive hoarseness but malignant transformation is rare
Carcinoma of the larynx
- 95% SCC
- Tobacco smoke & alcohol
- Hyperplasia to Invasive carcinoma spectrum
- Vocal cords, epiglottis or pyriform sinuses
- Hoarseness, pain, dysphagia & hemoptysis
- 65% -cured with surgery & radiotherapy
Salivary glands neoplasms
- Benign
o Pleomorphic adenoma/Mixed tumor (50 %)
o Warthin tumor (5-10 %)
o Oncocytoma (1 %)
o Basal cell adenoma (5-10 %)
o Canalicular adenoma (5-10 %)
- Malignant
o Mucoepidermoid carcinoma (15 %)
o Adenocarcinoma (NOS) (10 %)
o Acinic cell carcinoma (5 %)
o Adenoid cystic carcinoma (5 %)
o Malignant mixed tumor (3-5 %)
o SCC (1 %)
Pleomorphic adenoma
- Most common salivary gland neoplasm
- Mixed epithelial (ductal/glandular tissue) & mesenchymal differentiation
- Epithelial elements dispersed throughout a matrix of mucoid, myxoid, and
chondroid tissue
- Painless, slow-growing, mobile, discreet masses
- Local recurrences (Most likely prognosis)-25 % cases
- Malignant transformation-10 % after >15 years
Warthin tumor
- Papillary Cystadenoma Lymphomatosum
- Parotid
- 10 %- multifocal & bilateral
- Most common in smokers
- FNAC- scanty brown fluid, which microscopically consists of a mixture of lymphoid cells and epithelial cells with
abundant granular eosinophilic cytoplasm
- Micro: Glandular spaces lined by a double layer of epithelial cells around a dense lymphoid stroma; papillary tumor with
cleft like spaces lined by oncolytic cells
Mucoepidermoid Carcinoma
- Most common primary malignant salivary gland tumor
- Cords, sheets or cystic spaces of squamous, mucous and intermediate cells with mucus-filled vacuoles
Adenoid cystic carcinoma
- 50 % occur in minor salivary glands
- Small tumor cells with scanty cytoplasm
- Tubular or cribriform pattern
- Have a tendency to invade along perineural spaces, especially the facial nerve. This involvement can produce Bell’s palsy, which is characterized by flattening of
one side of the face
- Recurrent and invasive, eventually becoming metastatic

Disorders of the Esophagus


Symptoms of esophageal disease
- Dysphagia (Difficulty in swallowing) solids
o Symptom of an obstructive lesion
 Esophageal cancer
 Esophageal web
 Stricture
- Dysphagia for solids and liquids
o Symptom of a motility disorder
 Upper esophageal – striated muscle weakness
 Myasthenia gravis, stroke, dermatomyositis
 Lower esophageal – smooth muscle dysmotility
 Systemic sclerosis, CREST syndrome, Achalsia
- A 45-year-old woman presents with general discomfort and increasing tightness in the skin of her face. She reports intermittent pain in the tips of her fingers
when exposed to the cold. Physical examination shows “stone facies” and edema of the fingers and hands. Serologic tests for antinuclear and anti–Scl-70
antibodies are both positive. Which of the following gastrointestinal manifestations is expected in this patient? Dysphagia
- Odynophagia
o Pain on swallowing
o Causes- Gastric reflux; infectious esophagitis
- Heartburn
o Retro-sternal burning pain
o Associated with gastric reflux
- Hematemesis
o Vomiting blood
o Cause- Inflammation, ulceration, blood vessel rupture

Congenital and mechanical disorders


Tracheo-esophageal fistula
- Congenital connection b/w esophagus & trachea
- Proximal end of esophagus ends blindly
- Distal end arises from trachea
- Often discovered soon after birth due to aspiration
o Chemical pneumonia from milk reflux into lungs
o Stomach distension by air because air enters stomach from the tracheal fistula
o Maternal polyhydraminos : because swallowed amniotic fluid cannot be reabsorbed in the small intestine
o VATER syndrome : Vertebral anomalies, Anal atresia, TE fistula and Renal disease
- A 23-year-old primigravida gives birth at term to a boy infant. Ultrasound examination before delivery showed polyhydramnios. A single
umbilical artery is seen at the time of birth. The infant vomits all feedings, and then develops a fever and difficulty with respirations
within 2 days. A radiograph shows both lungs and the heart are of normal size, but there are pulmonary infiltrates and no stomach
bubble. What is the most likely diagnosis? Esophageal atresia
o An esophageal atresia is often combined with a fistula between the esophagus and trachea. Gastrointestinal obstruction in utero can lead to
polyhydramnios. The presence of a single umbilical artery suggests additional anomalies are present. Vomiting in an infant risks aspiration with
development of pneumonia.
Esophageal webs
- Web-like protrusions of esophageal mucosa into lumen.
- Annular narrowing in the upper third of the esophagus
- Clinical Findings:
o Dysphagia
o iron-deficiency anemia
- No evidence of inflammation or neoplasia
Plummer-Vinson syndrome/ Paterson-Brown-Kelly syndrome
- 90% cases-Middle-aged women
- Syndrome
o Esophageal webs (dysphagia for solids)
o Iron deficiency anemia
o Glossitis and cheliosis (angular stomatitis)
- Other features
o Koilonychia & achlorhydria
o Leukoplakia
o Increased risk of post-cricoid esophageal carcinoma
- A 20-year-old woman presents with a 2-year history of difficulty swallowing and increasing fatigue. A CBC shows iron-deficiency anemia. Upper endoscopy
reveals an annular narrowing in the upper third of the esophagus. A mucosal biopsy shows no evidence of infl mmation or neoplasia. Which of the following is
the most likely diagnosis? Esophageal web
Esophageal rings
- Concentric tissue plates protruding into distal esophageal lumen
- “A” ring: Above GEJ
- “B’’ring or Schatzki ring: At gastro-esophageal junction

Diverticula
- Out-pouchings of GI wall
- Types
o True
o False / pulsion diverticulum
- Typical symptoms
o Food regurgitation
o Dysphagia
o Neck mass
o Bad breath (halitosis)
o Bleeding
- Types of esophageal diverticula
o Zenker’s diverticulum (pulsion type)
 Above upper esophageal sphincter
 Defect in crico-pharyngeus muscle
 halitosis, regurgitation of undigested food, and change in the sound of voice
 A 65-year-old woman complains of a 4-month history of bad breath, regurgitation of undigested food, occasional aspiration of food, and
change in the sound of her voice. A barium swallow examination shows a posterior, midline pouch greater than 2 cm in diameter arising
just above the cricopharyngeal muscle. Which of the following is the most likely diagnosis? Zenker’s diverticulum
o Traction diverticulum
 Middle esophagus
 Abnormal motility or fibrosing mediastinal process
o Epiphrenic diverticulum
 Lower esophagus
Motor disorders
- Affect normal peristalsis of food and emptying of food into stomach (dysphagia for solids + liquids)
o Progressive systemic sclerosis/CREST
o Achalasia
Achalasia
- Neuromuscular disorder
o Definition
 Increased pressure and failure of LES to relax upon swallowing, and increased
intraesophageal pressure
 Manometric studies -Absence of peristalsis in proximal esophagus
o Etiology
 Primary/Unknown (most cases)
 Secondary/Acquired achalasia (and Hirchsprung’s disease)
 South America- Chaga’s disease (Trypanosoma cruzi)
 A 24-year-old woman living in eastern Bolivia has had increasing difficulty with swallowing
both liquids and solids for the past year. She has substernal discomfort from a feeling that
foods “get stuck” going down. On examination her BMI is 18. A barium swallow radiologically
shows marked esophageal dilation. An endoscopic biopsy is obtained and microscopically
shows reduced ganglion cells in myenteric plexus along with lymphocytic infiltration. Which
of the following organisms is most likely infecting this woman? Trypanosoma cruzi
o Pathogenesis
 Absent ganglion cells in myenteric plexus
 normally secrete VIP and NO (which relaxes LES)
- A 22-year-old woman has had multiple episodes of aspiration of food associated with difficulty swallowing during the past year. On auscultation of her chest,
crackles are heard at the base of the right lung. A barium swallow shows marked esophageal dilation above the level of the lower esophageal sphincter. A biopsy
specimen from the lower esophagus shows an absence of the myenteric ganglia. What is the most likely diagnosis? Achalasia
- A 35-year-old man complains of diffi culty swallowing and a tendency to regurgitate his food. Endoscopy does not reveal any esophageal or gastric abnormalities.
Manometric studies of the esophagus show a complete absence of peristalsis, failure of the lower esophageal sphincter to relax upon swallowing, and increased
intraesophageal pressure. Which of the following is the most likely diagnosis? Achalasia
Classic “Bird’s Beak” of Achalasia
- Barium Swallow
o “Bird-beak” sign
 Tortuous, widened, food-filled esophagus with a sigmoid form.
- Clinical findings:
o Progressive dysphagia to solids & liquids, nocturnal regurgitation,
aspiration
- Gross
o Esophageal dilatation proximal to LES
- Microscopy
o Loss of ganglion cells in myenteric plexus in the body of esophagus
o Rx: Nitrates, Calcium channel blockers (partial relief), Laparoscopic myotomy
o Pneumatic balloon dilatation
o Botox inhibits LES Cholinergic neurons
- Increased risk of esophageal squamous cell carcinoma (5%)
- A 44-year-old woman has had increasing difficulty swallowing liquids and solids for the past 6 months. On physical examination, her fingers have reduced
mobility because of taut, nondeforming skin. A barium swallow shows marked dilation of the esophagus with “beaking” in the distal portion, where there is
marked luminal narrowing. A biopsy specimen from the lower esophagus shows prominent submucosal fibrosis with little inflammation. Which of the following is
most likely to produce these findings? Systemic sclerosis
Lacerations (Mallory-Weiss syndrome)
- Longitudinal mucosal tears at GEJ
- Cause: Excessive vomiting with failure of LES relaxation due to alcoholism
- Clinical Finding: Massive hematemesis
- Complication: Boerhaave syndrome: esophageal rupture (rare)
- A 30-year-old man has sudden onset of hematemesis after a weekend in which he consumed large amounts of alcohol. The bleeding
stops, but he has another episode under similar circumstances 1 month later. Upper gastroesophageal endoscopy shows longitudinal
tears at the gastroesophageal junction. What is the most likely mechanism to cause his hematemesis? Vomiting
Boerhaave’s syndrome
- Transmural/full-thickness perforation of distal esophagus due to sudden rise in intraluminal esophageal pressure due to neuromuscular
incoordination
- Causes: Endoscopy (75% cases), retching, excessive vomiting in eating disorders like bulimia
- Complications
o Pneumo-mediastinum: Air dissects into subcutaneous tissue
o Pleural effusion
o Gastric juice in pleural space
- A 53-year-old man consumes a very large meal, washed down with considerable alcohol. The ensuing discomfort prompts him to take an emetic, but soon
afterward he develops lower chest pain. Physical examination reveals crepitus in subcutaneous tissue over his chest along with tachycardia and tachypnea.
Which of the following abnormalities of the esophagus is most likely present in this man? Rupture
Esophageal varices
- Definition
- Dilated submucosal veins in lower 1/3rd of esophagus, secondary to portal HT
- Causes
o Alcoholic cirrhosis (90%)
o Hepatic Schistosomiasis
- Clinical findings:
o Asymptomatic
o Massive hematemesis on rupture
- Complication: Potentially fatal hemorrhage
- In Portal Hypertension, the portal blood flow is diverted through stomach veins into the plexus of esophageal
subepithelial/submucosal veins and the resulting increase in pressure in the esophageal plexus produces dilated tortuous vessels (varices)
- Variceal hemorrhage subsides spontaneously in only 50% of cases, endoscopic injection of thrombotic agents (sclerotherapy)
or balloon tamponade is often required, 20% to 30% of patients die during the first episode
- A 51-year-old man has sudden onset of massive emesis of bright red blood. On physical examination, his temperature is
36.9Åã C, pulse is 103/min, respirations are 23/min, and blood pressure is 85/50 mm Hg. His spleen tip is palpable. Laboratory
studies show a hematocrit of 21%. The serologic test result for HBsAg is positive. He has had no prior episodes of
hematemesis. The hematemesis is most likely to be a consequence of which of the following? Esophageal varices
- A 58-year-old woman is brought to the emergency department 4 hours after vomiting blood and experiencing bloody stools.
The patient was diagnosed with alcoholic cirrhosis 2 years ago. The patient subsequently goes into shock and expires. The
histologic appearance of the esophagus at autopsy is shown in the image. Which of the following is the most likely underlying
cause of hematemesis and hematochezia in this patient?
Portal hypertension

Esophagitis
Gastroesophageal Reflux Disease (GERD)
- Definition
o Relaxation of LES with regurgitation of gastric secretions →inflammation of distal esophagus
- Predisposing factors
o Sliding hiatal hernias[80%]
o Pregnancy
o Peptic ulcer disease
o Factors lowering LES tone- ß-adrenergics, alcohol, smoking, caffeine, fatty foods
o A 45-year-old man presents with long-standing heartburn and dyspepsia. An X-ray film of the chest shows a retrocardiac, gas-filled structure. This
patient most likely has which of the following conditions? Hiatal hernia
- Presentation
o Usually > 40 years
o Dysphagia
o Heartburn (esp. at night)
o Regurgitation of gastric contents into mouth
- Complications
o Bleeding
o Hematemesis
o Melena
o Stricture
o Barret esophagus
- Reflux esophagitis - morphology
o Gross
 Mucosal redness, erosions and/or ulceration
o Microscopic
 Elongation of lamina propria papillae
 Basal layer hyperplasia
 Eosinophils +/- other inflammatory cells
- A 16-year-old boy who is receiving chemotherapy for acute lymphoblastic leukemia has had pain for 1 week when he swallows food. Physical examination shows
no abnormal findings. Upper gastrointestinal endoscopy shows 0.5- to 0.8-cm mucosal ulcers in the region of the mid to lower esophagus. The shallow ulcers are
round and sharply demarcated, and have an erythematous base. Which of the following is most likely to produce these findings? Gastroesophageal Reflux
Disease
- A 57-year-old woman has had burning epigastric pain after meals for more than 1 year. Physical examination shows no abnormal findings. Upper gastrointestinal
endoscopy shows an erythematous patch in the lower esophageal mucosa. A biopsy specimen shows basal zone squamous epithelial hyperplasia, elongation of
lamina propria papillae, and scattered intraepithelial neutrophils with some eosinophils. Which of the following is the most likely diagnosis? Reflux esophagitis
- A 50-year-old obese man (BMI = 32 kg/m2) comes to the physician complaining of indigestion after meals, bloating, and heartburn. Vital signs are normal. A CT
scan of the abdomen reveals a hiatal hernia of the esophagus. Endoscopic biopsy shows thickening of the basal layer of the squamous epithelium, upward
extension of the papillae of the lamina propria, and an increased number of neutrophils and lymphocytes. Which of the following is the most likely diagnosis?
Reflux esophagitis
Viruses
- Infectious esophagitis, in AIDS patients
o HSV type I (25% of all cases)
o CMV - Basophilic intranuclear inclusions
o Fungi (Candida) [elevated white plaques on endoscopy]; yeasts and
pseudohyphae
 A 30-year-old man with AIDS complains of severe pain on
swallowing. Upper GI endoscopy shows elevated, white
plaques on a hyperemic and edematous esophageal mucosa.
Which of the following is the most likely diagnosis? Candida esophagitis
o C/F: Odynophagia
o Herpes simplex virus: Endoscopy Vesicles and Punched out ulcers. Bx: multinucleated
giant cells with ground glass intra-nuclear inclusions.

- CMV Esophagitis
o Usually immunocompromised patients
o Deep ulcers
o Prominent intranuclear basophilic inclusions surrounded by clear halo as well as
cytoplasmic inclusions
Normal esophagus VS Barret’s esophagus

Barret’s Esophagus
- Definition
o Distal squamous mucosa is replaced by intestinal metaplastic (glandular) columnar
epithelium (GOBLET CELL) due to prolonged injury
- Cause
o Gastro-esophageal reflux disease (GERD) (5 to 15% patients)
- Gross:
o Irregular GEJ with tongues of red velvety granular mucosa extending up into the esophagus
- Criteria for diagnosis
o Endoscopic evidence: Of columnar epithelial lining above GEJ
- Barrett esophagus – morphology
o Definitive diagnosis is made when the columnar mucosa contains intestinal goblet cells
o Histologic evidence
 Of intestinal metaplasia in biopsy specimens from columnar epithelium.
- A 54-year-old man with a long history of indigestion after meals and “heartburn” presents with upper abdominal pain. He was
treated with proton-pump inhibitors for gastroesophageal reflux 3 years previously. An endoscopic biopsy of the lower esophagus is
shown in the image. Which of the following best describes these pathologic findings? Glandular metaplasia
- Complications
o Ulceration with stricture formation
o ↑sed risk of dysplasia and adeno-carcinoma
o Areas of dark red friable mucosa represent BE. Polypoid mass on biopsy proved to be an
adenocarcinoma
o Barrett esophagus - sequelae
 Ulceration, Bleeding, Stricture
 Adenocarcinoma

Esophageal Carcinoma
Squamous cell carcinoma (SCC)
- Epidemiology
o Most common type of esophageal cancer in the world, but not in the United States
o Males > females; Age >45 yrs
o African Americans > Caucasians
- Risk factors
o Heavy smoking and alcohol use
o Achalasia
o Plummer-Vinson syndrome
o Prior lye(caustic soda) ingestion
o Genetic factors: p16/INK4 tumor suppressor gene and EGFR, p53 in 50% of esophageal cancers
- Pathology
o Gross
 Mid-esophagus (50%)
 Polypoidal appearance
o Spread
 First spreads beneath mucosa
 Local spread by lymphatics → regional lymph nodes
 Systemic spread → liver, lungs, adrenals
 5-year survival: <5%
- Squamous cell carcinoma – morphology
o Mucosal epithelial dysplasia -> carcinoma in situ -> invasive cancer
 1. Polypoid exophytic masses
 2. Necrotizing ulcerations
 3. Diffuse infiltrative neoplasms
o Clinical findings:
 Asymptomatic until late
 Progressive dysphagia for solids
 Weight loss and anorexia
 Bleeding, hoarseness or cough (advanced cancers)
o Diagnosis
 Endoscopy and biopsy
o Prognosis- Poor
- A 73-year-old man with a history of chronic alcoholism has had increasing difficulty swallowing and has noticed a 3-kg weight loss over
the past 2 months. On physical examination, there are no remarkable findings. Upper gastrointestinal endoscopy shows a 3-cm
ulcerative mass in the midesophagus that partially occludes the esophageal lumen. Esophagectomy is performed; the gross
appearance of the lesion is shown in the figure. Which of the following is most likely to be seen on microscopic section of this mass?
Squamous cell carcinoma
- A 65-year-old woman presents with a 3-month history of diarrhea and abdominal pain. She has lost 9 kg (20 lb) in the past
6 months. The patient had two benign colonic polyps removed 3 years ago. Laboratory studies reveal mild iron-deficiency
anemia, and stool specimens are positive for occult blood. Sigmoidoscopy demonstrates an ulcerated mass, and a biopsy
shows malignant glands. A segment of the colon is resected, and the surgical specimen is shown in the image. Based on
current models of colonic carcinogenesis, which of the following genes was most likely mutated in the transition from
benign adenoma to carcinoma in this patient? p53

Adenocarcinoma of esophagus
- More common than SCC in the United States
- Caucasians > African Americans
- Arises in distal esophagus
- Associated with Barret esophagus and dysplasia
- Microscopic: mostly mucin-producing glandular tumors showing intestinal-type
features
- Mutations in p53 / alterations in HER-2/NEU and β-catenin
- Adenocarcinoma arising in Barrett esophagus is more common in whites than in
blacks
- Prognosis- Poor
- Diagnosis of Carcinoma : imaging techniques and endoscopic biopsy
- A 55-year-old man has had increasing difficulty swallowing during the past 6 months. There are no significant
findings on physical examination. Upper gastrointestinal endoscopy shows areas of erythematous mucosa 3 cm above the Z-line. A biopsy specimen from the
lower esophagus has changes in the mucosal epithelium illustrated in the figure. Which of the following complications is most likely to occur as a consequence of
this patient’s condition? Adenocarcinoma
- A 68-year-old man from Birmingham, England, has had “heartburn” and substernal pain after meals for 25 years. For
the past year, he has had increased pain with difficulty swallowing both liquids and solids. On physical examination,
there are no remarkable findings. Upper gastrointestinal endoscopy shows an ulcerated lower esophageal mass that
nearly occludes the lumen of the esophagus. A biopsy specimen of this mass is most likely to show which of the
following neoplasms? Adenocarcinoma
- A 70-year-old woman presents with difficulty swallowing and a 9-kg (20-lb) weight loss over the past several months.
Endoscopy reveals irregular narrowing of the lower third of the esophagus. A biopsy shows markedly atypical cuboidal
cells lining irregular gland-like structures. Which of the following is the most likely diagnosis? Adenocarcinoma
Carcinoma Esophagus
- Plummer Vinson syndrome---SCC - Nitrosamines containing
- Achalsia/Alcohol/Aspergillus---SCC foods---SCC
- Chemical Esophagitis---SCC - Barret esophagus--- Adenocarcinoma

Disorders of the Stomach 1


- Pyloric Stenosis
o Definition
 Congenital stenosis of pylorus
 Marked muscular hypertrophy of pyloric sphincter→ gastric outlet obstruction
o Males > females
o Clinical Course:
 Not present at birth but occurs over ensuing 2-4 wks
 Clinical findings
 Projectile vomiting of non-bile-stained fluid 2-4 wks after birth
 Visible peristalsis
 Palpable oval abdominal mass
 Hypertrophied pylorus is palpated in epigastrium.
- A 23-year-old woman, G2, P1, gave birth at term to a boy of normal weight and length following an uncomplicated pregnancy. The infant initially did well, but at
6 weeks, he began feeding poorly for 1 week, and his mother noticed that much of the milk he ingested was forcefully vomited within 1 hour. Now, on physical
examination, the infant is afebrile, and there are no external anomalies. A midabdominal mass is palpable. Bowel sounds are active. The medical history indicates
that both the mother and her first child had the same illness during infancy. Which of the following conditions is most likely to explain these findings? Pyloric
- A 3-week-old boy is brought to the physician by his parents, who report that he vomits forcefully immediately after nursing. Physical examination reveals an
“olive-like” palpable mass and visible peristaltic movements within the infant’s abdomen. What is the most likely cause of projectile vomiting in this infant?
Congenital pyloric stenosis
General Definitions
- Erosion - superficial defect in mucosa limited to lamina propria
- Ulcer - Breach in mucosa of alimentary tract extending through muscularis mucosa into submucosa or deeper
Acute gastritis
- An acute, transient inflammatory process (hemorrhagic / erosive)
- Erosions do not extend beyond mucosa
- Figure top: shows punctate erosions
- Etiology
o NSAID use (most common)
o Inhibits PG synthesis and its ability to produce and maintain bicarbonate-rich mucous barrier
- Other causes
o Excessive alcohol
o Heavy smoking
o Infection (eg. CMV in HIV patients)
o Uremia
o CNS injury (Cushing’s ulcers)
o Burns (Curling’s ulcers)
- Pathology
o Gross
 Edema and hyperemia
 Hemorrhage ±
o Microscopy
 Neutrophils invade the epitheliun
 Superficial epithelial sloughing (erosion)
- Clinical Features
o Asymptomatic
o Pain Epigastric pain
o Nausea, and vomiting
o Bleeding, Massive hematemesis, Melena, Potentially fatal blood loss
- A 72-year-old man takes large quantities of nonsteroidal anti-inflammatory drugs (NSAIDs) because of chronic degenerative arthritis of the hips and knees. Over
the past 2 weeks, he has had epigastric pain with nausea and vomiting and an episode of hematemesis. On physical examination, there are no remarkable
findings. A gastric biopsy specimen is most likely to show which of the following lesions? Acute gastritis
- A 54-year-old, previously healthy man sustained an extensive thermal burn injury involving 70% of the total body surface area of his skin. He was hospitalized in
stable condition. Three weeks after the initial burn injury, he developed melanotic stools. His blood pressure dropped to 80/40 mmHg, and his hematocrit
declined to 18%. Where are gastrointestinal ulcerations most likely to be found in this man? Stomach
- A 50-year-old woman with long-standing rheumatoid arthritis complains of weakness and fatigue. She states that her stools have recently become black after
taking a new nonsteroidal anti-inflammatory drug (NSAID). Gastroscopy shows numerous superficial, bleeding mucosal defects. Which of the following is the
most likely diagnosis? Acute erosive gastritis
Chronic Gastritis
- Symptoms are less severe than acute gastritis but persistent.
- Nausea, upper abdominal discomfort
- Sometimes vomiting BUT hematemesis uncommon (unlike peptic ulcer).
- Etiopathogenesis
o Chronic infection by H. pylori (Most Common)
o Immunologic (autoimmune) [<10%]
o In association with pernicious anemia
o Toxic- Alcohol, cigarette smoking
o Chronic bile reflux
o Radiation
o Granulomatous conditions (Crohn’s disease)
- Auto-immune [<10%] Body-fundic predominant
- H. pylori-related Antral-predominant
H Pylori Gastritis
- Predominantly antral gastritis with high acid production despite hypogastrinemia.
o [Only in few cases, involves gastric body & fundus (Pangastritis)  gastric atrophy  hypochlorhydria  intestinal metaplasia  adenocarcinoma]
- H Pylori colonizes. No invasion.
- Has flagella, Urease, Adhesins, Toxin (CagA)
- By some Mechanism increases acid production & decreases protective mechanisms.
- Most common type of chronic gastritis in US
- Microscopy Antral Gastritis (MC): Mainly in the Antral part of stomach
o H.pylori in mucus layer of surface epithelium
o Foci of acute inflammation  pit abscess.
o Intraepithelial neutrophils & subepithelial Plasma cells in LP
o Chronic inflammation & lymphoid follicles
o Microscopy PanGastritis (less common)
o Intestinal metaplasia [precursor for adenocarcinoma].
- Helicobacter pylori
o Gram-negative S-shaped rod (small, curved organism)
o >90% patients with chronic antral gastritis
o Fecooral transmission.
o Urease Ammonia
o Diagnosis: Silver stain will confirm
 Gastric biopsy  Rapid urease  Antibody
 Urea breath test test serologic tests
 Bacterial culture  DNA-based tests
Diseases Associated with Helicobacter pylori infection
- H Pylori in gastric mucosa causes
o Duodenal ulcer (almost all patients)
o Gastric ulcers (majority of cases)
o Chronic gastritis (90%)
o Peptic ulcer disease
o Gastric carcinoma
o Gastric MALT lymphoma
Auto-immune gastritis [MC cause for chronic atrophic gastritis]
- <10% cases of chronic gastritis. Decreased Acid production.
- Site of involvement
o body & fundus (acid-secreting part of stomach). Antral sparing.
- Associated conditions
o Hashimoto thyroiditis, Graves disease, and type 1 diabetes.
- Characteristics of Autoimmune Gastritis
o Auto-antibodies directed against parietal cells & IF
o Antral endocrine cell hyperplasia (G cell hyperplasia).
o Macrocytic (megaloblastic) anemia; vitamin B12 deficiency. pernicious anemia.
o Achlorhydria
o Increased risk of adenocarcinoma (10%) and carcinoid tumors.
- Gross
o Site: Body and fundic mucosa. Antral sparing.
o Mucosa: Long-standing atrophic disease Loss of rugal folds
- Microscopy
o Inflamm. infiltrate
 Lymphocytes/plasma cells, macrophages in lamina propria
o Lymphoid aggregates±
o Mucosal atrophy
 loss of glands/ parietal cells
o Intestinal metaplasia (presence of goblet cells)→ Dysplasia
- Lab investigations
o Hypochlorhydria or achlorhydria
o Hypergastrinemia [G cell hyperplasia in Antrum].
o Circulating autoantibodies to parietal cells (H+, K+ ATPase, Intrinsic
Factor).
- Complications
o Chronic atrophic gastritis (after 2-3 decades)
o Pernicious anemia
o Adenocarcinoma
o Carcinoid tumor
- A 51-year-old woman has been feeling increasingly tired for the past 7 months. There are no remarkable findings on physical examination. Laboratory studies
include hemoglobin, 9.5 g/dL; hematocrit, 29.1%; MCV, 124 μm3; platelet count, 268,000/mm3; and WBC count, 8350/mm3. The reticulocyte index is low.
Hypersegmented polymorphonuclear leukocytes are found on a peripheral blood smear. The serum gastrin is markedly increased. Antibodies to which of the
following are most likely to be found in this patient? Gastric H+,K+-ATPase
- A 34-year-old man presents with a 5-month history of weakness and fatigue. There is no history of drug or alcohol abuse. A CBC shows megaloblastic anemia and
a normal reticulocyte count. Further laboratory studies reveal vitamin B12 deficiency. Anemia
in this patient is most likely caused by which of the following? Autoimmune gastristis
- A 45-year-old woman presents with a 6-month history of fatigue and swelling in her neck. Physical examination shows a goiter. A CBC discloses megaloblastic
anemia and a normal reticulocyte count. Additionally, there is an elevated serum level of TSH and antithyroid antibodies. Needle aspiration of the left lobe of the
thyroid reveals benign follicular cells and numerous lymphocytes. Anemia in this patient is most likely caused by antibodies directed to which of the following
targets? Intrinsic factor
Chronic Atrophic Gastritis
- Definition
o Chronic mucosal inflammatory changes → mucosal atrophy and intestinal metaplasia
o Epithelial changes ---- dysplasia --- carcinoma
o MC cause is Autoimmune gastritis.

Disorders of the stomach- 2


Hypertrophic Gastropathy
- Hyperplasia of mucosal epithelial cells giant gastric mucosal folds
- Menetrier's Disease foveolar cell hyperplasia
- Hyperplastic hypersecretory gastropathy hyperplasia of partietal & chief cells
- Gastrc gland hyperplasia parietal cell hyperplasia, Z-E syndrome
Menetrier's Disease
- Hyperplasia of surface mucous cells & atrophy of fundic glands
- Cause : Excess of TGF α
- Gross Giant rugal folds
- Microscopy
o Massive hyperplasia of mucus secreting cells
o Replacement of parietal/chief cells
- Clinical findings:
o Middle-aged men
o Hypochlorhydria
o Protein losing gastropathy ---- hypoalbuminemia -> weight loss, diarrhea, peripheral edema
o ↑ed risk of gastric cancer
- A 42-year-old man presents with long-standing abdominal pain after meals, which is relieved by over-the-counter antacids. The patient has lost 9 kg (20 lb) in the
past year. Physical examination reveals peripheral edema and ascites. Laboratory studies show decreased serum albumin but normal serum levels of
transaminases and gastrin. Gross and microscopic examination of this patient’s stomach would most likely show which of the following pathologic changes?
Enlarged rugal folds
Zollinger-Ellison syndrome
- Small intestinal or Pancreatic gastrinomas producing gastrin.
- Increased acid secretion
- Clinical findings
o Multiple intractable peptic ulcers in duodenum, chronic diarrhea.
- A 45-year-old man describes burning epigastric pain 2 to 3 hours after eating. Foods, antacids, and over-the-counter medications provide no relief, and
prescribed inhibitors of acid secretion are only moderately effective. Recently, the patient noticed that his stools were black. Physical examination reveals
abdominal tenderness. The blood pressure is 120/80 mm Hg in the supine position and 90/50 mm Hg sitting up. The patient complains of lightheadedness upon
returning to a standing position. CBC shows a hemoglobin of 6.3 g/dL. Endoscopy reveals multiple gastric and duodenal ulcers. Epigastric pain and anemia are
most likely related to a neoplasm arising in which of the following anatomic locations? Pancreas
o Zollinger-Ellison syndrome is characterized by unrelenting peptic ulceration in the stomach or duodenum (or even proximal jejunum) by the action of
tumor-derived gastrin. Gastrin-producing neuroendocrine tumors (gastrinomas) usually arise in the pancreatic islets. Among islet cell tumors,
pancreatic gastrinomas are second in frequency only to insulinomas.
Peptic ulcer disease (PUD)
- PUD is most often associated with H pylori induced hyperchlorhydric chronic gastritis, which is present in 85% -100% of individuals with duodenal ulcers and 65
% with gastric ulcers.
- Definition
o Chronic, solitary lesions due to exposure to acid/peptic secretion
- Etiology
o H. pylori infection
o Chronic NSAID use
o Steroids, Smoking
- Common sites: Gastric antrum 1st part of duodenum
- Mucosa adjacent to peptic ulcer shows chronic gastritis. (to differentiate from acute gastritis/ stress ulcers)
- Pathogenesis -Imbalance b/w gastro-duodenal
o Mucosal defense mechanisms
 Gastric ulcers – Low mucosal resistance
o Damage from gastric acid/ pepsin
 Duodenal ulcers – Acid secretion is increased
- A 59-year-old man has had nausea and vomiting for 5 months. He has experienced no hematemesis. On physical
examination, there is no abdominal tenderness, and bowel sounds are present. Upper gastrointestinal endoscopy
shows erythematous areas of mucosa with thickening of the rugal folds in the gastric antrum. The microscopic
appearance of a gastric biopsy specimen with a Steiner silver stain is shown in the figure. Which of the following
factors is most likely responsible for this gastric mucosal pathology? Cytotoxin-associated gene A
- A 47-year-old woman with a lengthy history of heartburn and dyspepsia experiences sudden onset of abdominal pain. On physical examination, she has severe
mid epigastric pain with guarding. Bowel sounds are reduced. An abdominal plain film radiograph shows free air under the left leaf of the diaphragm. She is
immediately taken to surgery, and a perforated duodenal ulcer is repaired. Which of the following organisms is most likely to have produced these findings?
Helicobacter pylori
- A 35-year-old man has had epigastric pain for more than 1 year. The pain tends to occur 2 to 3 hours after a meal and is relieved if he takes antacids or eats more
food. He has noticed a 4-kg weight gain in the past year. He does not smoke and drinks 1 glass of Johannisberg Riesling daily. The result of a urea breath test is
positive, and a gastric biopsy specimen contains urease. He begins a 2-week course of antibiotics, but on day 4, he feels better and discontinues treatment. Three
weeks later, the epigastric pain recurs. If he does not seek further treatment, which of the following complications is he most likely to develop? Hematemesis
- A 40-year-old woman presents with a 2-month history of burning epigastric pain that usually occurs between meals. The pain can be relieved with antacids or
food. The patient also reports a recent history of tarry stools. She denies taking aspirin or NSAIDs. Laboratory studies show a microcytic, hypochromic anemia
(serum hemoglobin = 8.5 g/dL). Gastroscopy reveals a bleeding mucosal defect in the antrum measuring 1.5 cm in diameter. An endoscopic biopsy shows that the
lesion lacks mucosal lining cells and is composed of amorphous, cellular debris and numerous neutrophils. Which of the following is the most important factor in
the pathogenesis of this patient’s disease? Helicobacter pylori infection
- A 58-year-old woman suffers a massive stroke and expires. The stomach at autopsy is shown in the image. Prior to her death, this patient would most likely have
exhibited which of the following? Melena
o Peptic ulcers of the stomach and duodenum are estimated to afflict 10% of the population of Western industrialized countries at some time during
their lives. Peptic ulcers appear as punched out, rounded ulcers. Erosion through arteries causes bleeding and iron-defi ciency anemia. Melena refers
to black, tarry stools composed largely of blood from the upper digestive tract that has been processed by the action of gastric juices. Melena is
commonly seen in patients who suffer from chronic peptic ulcer disease.
- A 60-year-old man presents with epigastric pain after meals, with some nausea and vomiting. A burning sensation in the midepigastrium is relieved by antacids
and H2 antagonists. Upper endoscopy demonstrates paired ulcers on both walls of the proximal duodenum. Which of the following represents the most common
complication of this patient’s duodenal disease? Bleeding

Gastric hyperacidity drives PUD


- H pylori
- Parietal cell hyperplasia
- Loss of inhibition on gastrin release Eg: Zollinger Ellison syndrome
- Psychological stress
- Factors disrupting mucosal defence
o NSAID
o steroids
o smoking
- Clinical presentation
o Any age.
o Burning epigastric pain 1-3 hours after eating food
o Worse at night. Relieved by alkali / food
o Nausea, vomiting, bloating, belchingFrank hemorrhage / perforation
o Penetrating ulcers  Pain to back
- Sites of involvement
o Solitary lesions < 4 cm in diameter, in
o Duodenum (1st portion)
 Anterior part (most common)
 Posterior portion (perforation into pancreas)
o Stomach (antrum)
 Lesser curvature
o Other locations
 At the GEJ, in association with GERD or BE
 Margins of a gastro-jejunostomy
 Ileal Meckel’s diverticulum with ectopic gastric mucosa
o Gross- Ulcer
 Sharply punched-out defect
 Overhanging mucosal borders
 Smooth, clean ulcer bases
- Microscopy
o Sup. layer of necrotic fibrinoid debris 
Inflammation with neutrophils predominating
Granulation tissue Fibrous or collagenous scar
- Complications
o Perforation [Common in DU]
 65% of ulcer deaths.
 Free air under diaphragm
o Hemorrhage (25-30%)
 Massive hemetemesis
 25% of ulcer deaths
 GU erodes Left Gastric A
 DU erodes Gastroduodenal A
o Obstruction
 from edema or scarring
o Iron deficiency anemia
o Penetration into adjacent organs – pancreas
o Malignant transformation is unlikely
Bezoars
- Luminal concretions of indigestible ingested material
- Trichobezoar
o Hairball
o A 23-year-old woman with a history of an eating disorder complains of vomiting, nausea, and severe
abdominal pain. Physical examination shows abdominal distension and constipation. An X-ray film of
the abdomen reveals air-fluid levels and a hyperlucent shadow at the epigastric area. The material
obstructing the gastrointestinal tract is removed surgically and shown. Which of the following is the
most likely diagnosis? Trichobezoar
- Phytobezoar
o Derived from plant material

Disorders of the Stomach-3


Hyperplastic (inflammatory) polyps
- Non-neoplastic polyps
- 90% of gastric polyps
- Associated with chronic gastritis
- Gross
o Smooth
o Sessile or pedunculated
- Microscopy
o Epithelial cysts and tubules
o Inflammatory stroma
- A 52-year-old man notes nausea with abdominal discomfort after meals. On physical examination, there are no abnormal findings. Upper endoscopy is
performed, and there are three ovoid nodules in the fundus and antrum ranging from 0.3 to 1.2 cm in size. They have rounded, smooth surfaces. Biopsies are
taken and on microscopic examination there are irregular, cystically dilated and elongated foveolar glands. Which of the following treatment strategies is most
appropriate for his gastric lesions? Antibiotics
Adenomas
- 5-10% of gastric polyps Leiomyomas
- Incidence increases with age
- Proliferative, metaplastic & dysplastic epithelium
- 40% --- gastric adenocarcinoma
Leiomyomas
- Most common benign soft tissue tumors in GIT
- Most common location- stomach
- May ulcerate and bleed
Adenomas
Gastric Cancers
Gastric adenocarcinoma
- Epidemiology
o Common in Japan, chile Costa Rica, Eastern Europe.
o Endoscopic screening in Japan
o Decreasing in USA – Reduced use of smoked & salted food. Increased use of Green leafy vegetables & citrus fruits rich in antioxidants Vit C & E
- Pathogenesis
o Environmental Factors
 Diet
 Nitrites derived from nitrates (water, preserved food) .
 Smoked and salted foods, pickled vegetables, chili peppers
 Lack of fresh fruit and vegetables
o Low socio-economic status
o Cigarette smoking
- Host Factors
o Auto-immune gastritis
o Chronic gastritis - H. pylori
o Partial gastrectomy - Favors gastro-duodenal reflux
o Gastric adenomas
 40% harbor cancer at time of diagnosis
 30% have adjacent cancer at time of diagnosis
o Barrett esophagus
o Atrophic gastritis with intestinal metaplasia
o PUD do not become cancer
- Genetic Factors
o Increased risk with blood group A
o Family history of gastric cancer
o Hereditary non-polyposis colon cancer syndrome
o Familial gastric carcinoma syndrome (E-cadherin mutation) (usually diffuse type)
- Clinical presentation
o Often (90%) asymptomatic until late
o Dyspepsia, nausea, dysphagia
o Weight loss and anorexia
o Epigastric abdominal pain ~ peptic ulcer
o Early satiety and pain after eating large meals
o Occult bleeding and iron deficiency anemia
- A 53-year-old woman has had nausea, vomiting, and midepigastric pain for 5 months. On physical examination, there are no significant
findings. An abdominal CT scan shows gastric outlet obstruction. Upper gastrointestinal endoscopy shows an ulcerated 2 Å~ 4 cm bulky
mass in the antrum at the pylorus. A urease test is positive. Which of the following neoplasms is most likely to be seen in a biopsy
specimen of this mass? Adenocarcinoma
- A 60-year-old man presents with an 8-week history of progressive weight loss, nausea, and upper abdominal pain that does not
respond to antacids or H2-receptor antagonists. Laboratory studies show iron-deficiency anemia. Gastroscopy reveals a crater-like,
ulcerated lesion in the antrum, with raised, irregular, and indurated margins. The patient undergoes partial gastrectomy and the
surgical specimen is shown in the image. Which of the following is the most likely diagnosis? Adenocarcinoma
- Gross
o Location
 Cardia(25%)
 Body and fundus (25%)
 Pylorus, antrum (50-60%)
o Flat /polypoid / excavated lesions
o Ulcer > 3 cm, heaped-up margins, necrotic ulcer base
o
- Morphologic types of Carcinoma Stomach

Fungating Ulcerating Diffuse

Diffuse type
- Linitis plastica
- Thickened leather-bottle-like stomach due to excess desmoplasia
- A 58-year-old woman presents with a 2-month history of abdominal discomfort and dark stools. Physical examination shows pallor but no evidence of jaundice.
Laboratory studies disclose a microcytic, hypochromic anemia, with a hemoglobin level of 6.7 g/dL. A barium swallow radiograph reveals a “leather bottle”
appearance of the stomach. Microscopic examination shows diffusely infi ltrating malignant cells, many of which are “signet ring” cells, in the stomach wall.
Which of the following is the most likely diagnosis? Linitis plastica
Classification of gastric carcinoma
- Based on
o Depth of invasion
o Macroscopic growth pattern
o Histologic subtype
Based on depth of invasion
- Early gastric carcinoma
o Lesion confined to mucosa/submucosa
- Advanced gastric carcinoma
o Extension beyond submucosa into muscular wall
Based on macroscopic growth pattern
- Exophytic
o Protrusion of a tumor mass into the lumen
- Flat / depressed
o No obvious tumor mass within the mucosa
- Excavated
o A shallow or deeply erosive crater is present in the stomach wall
-

Based on histologic subtype


- Intestinal type
o Gland forming adenocarcinoma Intestinal type
- Diffuse type
o Not related to H. pylori
o Diffuse infiltration of stomach by poorly differentiated tumor cells
o Signet-ring cells carcinoma (Linitus Plastica)
o Nucleus is displaced to periphery by intra-cellular mucin
o A 67-year-old woman has experienced severe nausea, vomiting, early satiety, and a 9-kg weight loss over the past 4 months. On physical examination,
she has muscle wasting. Upper gastrointestinal endoscopy shows that the entire gastric mucosa is eroded and has an erythematous, cobblestone
appearance. An abdominal CT scan shows that the stomach is small and shrunken. Which of the following is most likely to be found on histologic
examination of a gastric biopsy specimen? Signet ring cell adenocarcinoma

Signet ring cells


D/D of Gastric ulcers
Metastasis
- Virchow (sentinel) node
o Left supra-clavicular LN GIST
- Sister Joseph Mary nodule
o Subcutaneous nodule in peri-umbilical region
- Krukenberg tumor
o Spread to ovary
- Paraneoplastic skin lesions
o Acanthosis nigricans
o Multiple outcroppings of seborrheic keratoses (Leser-Trelat sign)
Gastrointestinal stromal tumor (GIST)
- Tumor composed of cells having elongated, spindle-shaped nuclei.
- The tumor cells stain positively with CD117, but negatively with both desmin and S-100.
- Mutations of the C-KIT gene  Tyrosine kinase activation.
- C/F: Asymptomatic, mucosal ulcer  bleeds
- Rx: Surgery ; Imatinib mesylate
- A 61-year-old man with increasing fatigue, early satiety, and nausea for 5 months vomited dark granular material yesterday.
Endoscopy reveals a large ulcerated mass in the gastric fundus. Biopsies are taken and microscopically the mass is composed
of spindle cells that are positive for c-Kit with immunohistochemical staining. Mitoses are frequent. Gastrectomy is
performed, and the 10-cm circumscribed mass arises from the gastric wall. Which of the following therapies is most likely to
be a useful adjunct in treatment of his disease? Imatinib
- A 55-year-old woman complains of upper gastrointestinal pain and tarry stools. Upper endoscopy shows a firm, smooth,
yellowish submucosal ulcerated mass in the stomach. Gastroscopic biopsy reveals spindle cells with vacuolated cytoplasm.
The mass is removed, and the surgical specimen is shown in the image. Which of the following is the most likely diagnosis?
Gastrointestinal stromal tumor
Primary malignant lymphomas
- Stomach – Most common extra-nodal site
- Origin- Mucosa-associated-lymphoid tissue (MALT)
- Grade
o High-grade, large-cell immunoblastic lymphomas of B-cell origin
o Low-grade B-cell lymphomas (MALTomas) more likely associated with H.pylori
- A 52-year-old man has had a 4-kg weight loss and nausea for the past 6 months. He has no vomiting or diarrhea.
On physical examination, there are no remarkable findings. Upper gastrointestinal endoscopy shows a 6-cm area
of irregular, pale fundic mucosa and loss of the rugal folds. A biopsy specimen shows a monomorphous infiltrate
of lymphoid cells microscopically. Helicobacter pylori organisms are identified in mucus overlying adjacent
mucosa. Cytogenetic analysis shows t(11;18)(q21;q21). He receives antibiotic therapy for H. pylori, and the
repeat biopsy specimen shows a resolution of the infiltrate. What is the most likely diagnosis? Mucosa-
associated lymphoid tissue tumor
- A 56-year-old woman comes to the physician after noticing multiple lumps in her neck. Physical examination
shows enlarged and nontender supraclavicular lymph nodes. Upper endoscopy discloses thickening of the gastric
mucosa, without an obvious tumor. The results of gastric biopsy are shown in the image. Which of the following is the most likely diagnosis? Gastric lymphoma

Disorders of the Small Bowel


Omphalocele
- Ventral abdominal wall defect involves region of umbilical cord. A thin membrane covers herniated abdominal contents (loops of bowel
seen under the membrane). Since bowel mainly developed outside abdominal cavity, it is malrotated and cavity is not properly formed
(too small).
Duodenal atresia
- Asso. with Down syndrome
- H/O maternal polyhydramnios
- Clinical findings: Vomiting of bile-stained material at birth
- Abdominal X-ray: “Double-bubble” sign
Bowel obstruction Duodenal atresia:
- Mechanical Obstruction
o Hernias, internal or external
“Double bubble”
o Adhesions ( Most common cause of small bowel obstruction)
o Intussusception
o Volvulus (twisting of bowel)
o A 57-year-old man from Innsbruck, Austria, goes to the emergency department because of increasing abdominal pain with distention that developed
over the past 24 hours. On physical examination, there is diffuse abdominal tenderness. The abdomen is tympanitic, without a fluid wave, and bowel
sounds are nearly absent. There is a well-healed, 5-cm transverse scar in the right lower quadrant of the abdomen. There is no caput medusa. A stool
sample is negative for occult blood. An abdominal plain film shows dilated loops of small bowel with air-fluid levels, but there is no free air. At
laparotomy, the surgeon notices a 20-cm portion of reddish black ileum that changes abruptly to pink-
appearing bowel on distal and proximal margins. His medical history is significant only for an appendectomy at
age 25 years. Which of the following is most likely to have produced his findings? Adhesions
- Other Less Frequent Conditions
o Tumors o Congenital bands
o Inflammatory strictures o Meconium in cystic
o Obstructive gallstones, fecaliths, fibrosis
o foreign bodies o Imperforate anus
o Congenital stricture, atresias
- Clinical Findings : Abdominal pain, distension, vomiting, constipation
o Multiple dilated loops of gas filled bowel. Air fluid levels.
- Types
o Hernias: Inguinal and femoral canals, umbilicus,
surgical scars, retroperitoneum
 External herniation, Strangulation
o Adhesions : Surgical procedures, infection,
endometriosis – peritonitis
o Intussusception : telescoping of a proximal segment
of bowel into the immediately distal segment.
Intestinal obstruction and infarction of the trapped
segment.
o Volvulus : Bowel twists around mesenteric root
producing obstruction and strangulation, Sigmoid
colon (elderly) Cecum ( young adults) – most
common site
Volvulus
- Loop of bowel twists around its own mesentery → intestinal obstruction and infarction & peritonitis
- Associated with intestinal malrotation
- Locations: Sigmoid colon, cecum, small bowel
- Complications: Infarction, peritonitis
Intussusception Volvulus
- Definition
o Telescoping of a proximal segment (terminal ileum) of bowel into distal segment (cecum)
- C/F
o Infants /children. Weaning [adenovirus]
o Intestinal obstruction, Bloody stool, colicky abdominal pain
o Oblong/Sausage-shaped mass
o “Currant-jelly” stools [mucosal infarction & blood clots] Intussusception
o Rx: Barium enema, surgery
- Complication
o Infarction of intussuscepted segment
- Figure Bottom: A cross section through the area of the obstruction shows “telescoped” small intestine surrounded by dilated large intestine.
- An 11-month-old, previously healthy infant has not produced a stool for 1 day. The mother notices that the infant’s abdomen is distended. On physical
examination, the infant’s abdomen is very tender, and bowel sounds are nearly absent. An abdominal plain film
radiograph shows no free air, but there are distended loops of small bowel with air-fluid levels. Which of the following is
most likely to produce these findings? Intussusception
- A 2-year-old boy is brought to the emergency room with a 48-hour history of nausea, vomiting, and abdominal discomfort.
Physical examination reveals right lower quadrant guarding. Ultrasound examination of the abdomen reveals a 2-cm mass
in the right iliac fossa. A segment of the small intestine is removed (shown in the image). Which of the following best
describes this pathologic finding? Intussusception
Ischemic Bowel Disease
- Mucosal infarction: no deeper than the muscularis mucosae.
- Mural infarction: mucosa and submucosa.
- Transmural infarction: involving all visceral layers
- Causes
o Transmural lesions: Usually due to occlusion of SMA(Sup.Mesen.art)
o Mural and mucosal ischemia: hypoperfusion states (shock)
o Arterial thrombosis
o Arterial embolism Infarcted
o Venous thrombosis
o Nonocclusive ischemia small bowel
o Atherosclerosis
o Aortic aneurysm
o Hypercoagulable states, OCPs
o Embolisation of cardiac vegetations / aortic atheromas
- Figure: Infarcted small bowel, secondary to acute thrombotic occlusion of the superior mesenteric artery
- A 61-year-old man has had severe abdominal pain and bloody diarrhea for the past day. On physical examination, his abdomen is diffusely tender, and bowel
sounds are absent. Abdominal plain films show no free air. Laboratory studies show a normal CBC and normal levels of serum amylase, lipase, and bilirubin. His
Hgb A1c is 10%. He develops shock. A year ago he had an acute myocardial infarction. Which of the following lesions is most likely to be found in this man?
Intestinal infarction
- A 70-year-old man is rushed to the emergency room complainingof severe abdominal pain and rectal bleeding of 2 hours in duration. He has a history of coronary
artery disease. Bowel sounds are absent on physical examination. A CT scan of the abdomen shows distention of the stomach and air-fluid levels in the small
bowel. Abdominal pain and bleeding in this patient most likely involved acute occlusion of which of the
following arteries? Superior mesenteric artery
Acute ischemia involving small bowel
- Acute mesenteric ischemia (50%)
o EMBOLISM from the left side of the heart to the SMA
 (Atrial fibrillation -most common predisposing arrhythmia)
o THROMBOSIS of the SMA
- Nonocclusive ischemia (25% of cases)
o Hypotension secondary to heart failure (most common)
- Clinical Features: Older individuals with preexisting cardiac or valvular disease.
o Sudden onset diffuse abdominal pain (pain disproportionate to physical findings), nausea, vomiting, bloody diarrhea.
o Absent bowel sounds (ileus), No rebound tenderness (peritonitis) early in infarction, Muscle spasm of Anterior abdominal wall  board like rigidity.
Profound neutrophilic leukocytosis, Positive stool guaiac
- Treatment: Surgery for embolic disease
o Anticoagulation and surgery if necessary for thrombotic disease
- A 71-year-old woman with a history of rheumatic heart disease is hospitalized with severe congestive heart failure. Four days after admission, she develops
cramping lower abdominal pain. On physical examination, she is afebrile. The abdomen is distended and tympanitic, without a fluid wave, and bowel sounds are
absent. A stool sample is positive for occult blood. An abdominal plain film shows no free air. Colonoscopy shows patchy areas of mucosal erythema with some
overlying tan exudate in the ascending and descending colon. No polyps or masses are found. What is the most likely diagnosis? Ischemic colitis
Malabsorption
- Suboptimal absorption of fats, fat-soluble and other vitamins, proteins, carbohydrates, electrolytes, minerals, and water.
o Intraluminal digestion / Mucosal absorption / Nutrient delivery
o The malabsorptive disorders most commonly encountered in the United States are pancreatic insufficiency, celiac disease, and Crohn disease
- C/F - Weight loss, anorexia, malnutrition, abdominal distension, borborygmi, muscle wasting.
o Steatorrhea: Excess fecal fat & bulky, frothy, greasy, yellow or clay coloured stools. Floats on water.
Celiac sprue/ disease
- Synonym
o Gluten-sensitive
o enteropathy Loss of villi
o Non-tropical sprue
- Figure: Loss of normal folds in small intestinal mucosa
- Definition
o Hypersensitivity to gluten (and gliadin)-wheat, barley, rye, oats.
o Loss of small bowel villi and malabsorption
o Commonest malabsorption syndrome in US.
- Serology: IgA Abs to transglutaminase, IgA/IgG Abs to deamidated gliadin,
o AntiEndomysial antibodies **
- Microscopy (duodenum & proximal jejunum)
o Loss of villi
o ↑ed surface intra-epithelial lymphocytes Marked villous
o ↑ed plasma c ells in lamina propria
o Marked villous atrophy (flattening of villi) atrophy
 Important feature of celiac disease on biopsy
o Crypt elongation, increased epithelial mitosis
- Figure Bottom: Celiac sprue – flattening villi
- Clinical presentation
o Presents in childhood with malabsorption
o Children
 Failure to thrive, abdominal distension, periodic diarrhea
o Adults [30-60 yrs]
 Weight loss, diarrhea, malnutrition, anemia.
 Increased risk of T-cell lymphoma
Flattening
o Associated with dermatitis herpetiformis villi
- Treatment: Dietary restriction of gluten and gliadin
- A 34-year-old woman is bothered by a low-volume, mostly watery diarrhea
associated with flatulence. The symptoms occur episodically, but they have been persistent for the past
year. She has experienced a 4-kg weight loss. She has no fever, nausea, vomiting, or abdominal pain. On
physical examination, there are no significant findings. A stool sample is negative for occult blood, ova,
and parasites, and a stool culture yields no pathogens. An upper gastrointestinal endoscopy is performed
and a biopsy specimen from the upper part of the small bowel shows severe diffuse blunting of villi and a chronic inflammatory infiltrate in the lamina propria.
Which of the following serologic tests is most likely to be positive in this patient? Antitransglutaminase antibody
- A 41-year-old woman has had diarrhea and fatigue with a 3-kg weight loss over the past 6 months. On physical examination, she is afebrile and has mild muscle
wasting, but her motor strength is normal. Laboratory studies show no occult blood, ova, or parasites in the stool. A biopsy specimen from the upper jejunum is
obtained, and microscopic findings are reviewed. The patient begins following a special diet with no wheat or rye grain products. The change in diet produces
dramatic improvement. Which of the following microscopic features is most likely to be seen in the biopsy specimen? Villous blunting and flattening
- A 4-year-old girl is brought to the physician because her parents noticed that she has been having pale, fatty, foul-smelling stools. The patient is at the 50th
percentile for height and 10th percentile for weight. Her symptoms respond dramatically to a gluten-free diet. Which of the following is the most likely diagnosis?
Celiac sprue
Tropical Sprue
- Definition
o Malbsorptive disease of unknown etiology
o Affects travelers to tropical regions (Caribbean and South America)
o C/F: wt loss, steatorrhea, anorexia, malaise following an episode of acute diarrhea.
- Microscopy ~ celiac sprue [flattening of villi, inflammation]
- Treatment: Antibiotics, vitamin B12, and folate

Whipple’s disease
o Rare infectious disease
o Involves many organs (small intestines, joints, LNs, lungs, heart and CNS)
o Arthritis, malabsorption, diarrhea, steatorrhea, wt loss
- Epidemiology : Caucasian males; 30-50 yrs
- Organism : Tropheryma whippelii (gram-positive and culture-resistant
actinomycete)
o Mucosal injury  impaired transport of nutrients
o Mesentric LNs show organism laden macrophages 
impaired lymphatic transport due to lymphatic obstruction.
- Treatment – Antibiotics
- Clinical presentation
o Fever, diarrhea, joint pain, gen. LN
o Malabsorption, weight loss
- Microscopy
o Blunting of villi in jejunum and ileum
o Lamina propria
o Foamy macrophages with PAS +ve, rod-shaped bacilli * (AFB neg, to diff from TB)
- A 36-year-old man presents with fever and painful joints for 2 weeks. Physical examination shows skin
pigmentation, glossitis, angular cheilitis, and generalized lymphadenopathy. The patient has lost 9 kg (20 lb) over
the past 6 months. He reports that his stools are pale and foul smelling. Blood cultures are negative. The patient is
started on antibiotic therapy and exhibits remarkable clinical improvement. Biopsy of the small intestine shows
marked distortion of the intestinal villi, and a periodic acid-Schiff stain reveals large, foamy macrophages fi lled
with glycoprotein-rich granules (shown in the image). Which of the following is the most likely diagnosis?
Whipple’s disease
- Which of the following cellular/biochemical mechanisms best explains the pathogenesis of malabsorption in the
patient described in above question? Impaired mucosal function
ENTEROCOLITIS (Diarrheal diseases)
- Increase in stool mass, stool frequency, or stool fluidity
- Daily stool production >250 gm, containing 70% to 95% water
- 14 L/day of fluid may be lost in severe cases
- Low-volume, painful, bloody diarrhea with mucus - dysentery
- Tenesmus: Painful, ineffective straining at stool
Enterocolitis (Types)
- Secretory diarrhea: secretion that is isotonic with plasma and persists during fasting
- Osmotic diarrhea: excessive osmotic forces exerted by luminal solutes that abate with fasting (lactase deficiency)
- Exudative diarrhea: purulent, bloody stools that persists on fasting; stools are frequent but may be small or large volume
- Malabsorption diarrhea: voluminous, bulky stools with increased osmolarity resulting from unabsorbed nutrients and excess fat (steatorrhea); it usually abates
on fasting
- A 21-year-old man has had increasingly voluminous, bulky, foul-smelling stools and a 7-kg weight loss for the past year. There is no history of hematemesis or
melena. He has some bloating, but no abdominal pain. On physical examination, there are no palpable abdominal masses, and bowel sounds are present. Which
of the following laboratory findings is most likely to be present on examination of his stool? Increased stool fat
E. coli
- Enterotoxigenic strains (ETEC) : small intestine, with histologic features similar to V. cholerae. Travellers diarrhoea.
- The Shiga toxin-producing strain (STEC) O157:H7 : right colon, with hemorrhage and ulceration. In children - may be followed by the hemolytic uremic
syndrome (most common strain of E. coli in North America)
- Contaminated ground beef.
- Enteroinvasive strains (EIEC): colon, histology similar to Shigella
- Enteroaggregative strains (EAEC) also affect the colon and show a "stacked brick" pattern of adherence in tissues
- A 59-year-old man with a lengthy history of chronic alcoholism has noticed increasing abdominal girth for the past 6 months. He has had increasing abdominal
pain for the past 2 days. On physical examination, his temperature is 38.2Åã C. Examination of the abdomen shows a fluid wave and prominent caput medusae
over the skin of the abdomen. There is diffuse abdominal tenderness. An abdominal plain film radiograph shows no free air. Paracentesis yields 500 mL of cloudy
yellow fluid. Gram stain of the fluid shows gram-negative rods. Which of the following is the most likely diagnosis? Spontaneous bacterial peritonitis
- A 30-year-old woman presents with 2 days of abdominal cramping and diarrhea. Her temperature is 38°C (101°F), respirations are 32 per minute, and blood
pressure is 100/65 mm Hg. Stool culture shows a toxigenic Escherichia coli infection. Which of the following best explains the pathogenicity of this organism in
this patient? Stimulation of fluid transport into the lumen of the intestine.

Pseudo-membranous colitis
- Acute colitis → Inflammatory pseudo-membranes in intestines
- Clostridium difficile *
o Toxin  cytoskeletal disruption  apoptosis
- Predisposing factor: Broad spectrum antibiotics (ampicillin and clindamycin, 3rd gen cepahalosporins )
- C/F: Diarrhea, fever, abdominal cramps
- Gross: Yellow-tan mucosal membranes
- Microscopy
o Superficial colonic necrosis
o Overlying pseudo-membrane
o Inflamm. exudate of neutro, mucin, fibrin, necrotic debris
- Diagnosis - C. difficile toxin assay in stool
- Treatment – Oral Vancomycin / metronidazole
- A 65-year-old woman is being treated in the hospital for pneumonia complicated by septicemia. She has required
multiple antibiotics and was intubated and mechanically ventilated earlier in the course. On day 20 of hospitalization,
she has abdominal distention. Bowel sounds are absent, and an abdominal radiograph shows dilated loops of small bowel suggestive of ileus. She has a low
volume of bloody stool that is positive for Clostridium difficile toxin. Laboratory studies show leukocytosis and hypoalbuminemia. At laparotomy, a portion of
distal ileum and cecum is resected. The gross appearance of the mucosal surface is shown in the figure. What is
the most likely diagnosis? Pseudomembranous enterocolitis
- A 53-year-old woman complains of acute diarrhea and severe abdominal pain. She was recently treated with
broad-spectrum antibiotics for community-acquired pneumonia. A CBC shows a WBC count of 24,000/μL. The
patient subsequently develops septic shock and dies. A portion of her colon is shown at autopsy. These findings
are typical of which of the following gastrointestinal diseases? Pseudomembranous colitis
Protozoal Infection Entamoeba
- Entamoeba histolytica –
o Flask-shaped ulcers histolytica
o Hepatic abscesses
- Entamoeba histolytica in the colon. Some organisms are ingesting red blood cells
- Giardia lamblia : normal to marked blunting of the villi, malabsorptive diarrhea
- Cryptosporidiosis : major cause of childhood diarrhea (20% cases in developing countries), Most common cause of
diarrhea in AIDS
- A study of children living in rural Malawi in Africa Reveals a high prevalence of iron deficiency anemia. Stool samples are
positive for occult blood. Pruritus of the skin of their feet as well as cough are additional findings in many of these
children. Which of the following parasitic infestations is the most likely cause for these findings? Ancylostoma
duodenale
- A 31-year-old woman had increasingly severe diarrhea 1 week after returning from a trip to Central America. Gross examination of the stools showed mucus and
streaks of blood. The diarrheal illness subsided within 4 weeks, but now she has become febrile and has pain in the right upper quadrant of the abdomen. An
abdominal ultrasound scan shows a 10-cm, irregular, partly cystic mass in the right hepatic lobe. Which of the following infectious organisms is most likely to
produce these findings? Entamoeba histolytica
Cryptosporidium
- Cryptosporidium parvum (partially acid fast positive oocysts) and Microsporidia are associated with immunocompromised
hosts (AIDS)
- Clinical findings:
o Watery diarrhea (resembling cholera) Cryptosporidium
Typhoid fever (enteric fever)
- Caused by S.typhi (human reservoir)
- Source of spread
o Food or contaminated water
- Anorexia, abdominal pain, nausea, vomiting & bloody diarrhoea followed by
- Short asymptomatic phase  bacteremia & flu like fever.
- Blood culture positive in febrile phase.
- Complication : Intestinal perforation
- Chronic carrier state (persistence of organism 1 year post-infection), usually in gallbladder
- In an epidemiologic study of infections of the gastrointestinal tract, cases of patients living in Haiti from whom definitive cultures were obtained are analyzed for
clinical and pathologic findings that may be useful for diagnosis. A group of patients is identified who initially had abdominal pain and diarrhea during week 1 of
their illness. By week 2, these patients had splenomegaly and elevations in serum AST and ALT levels. By week 3, they were septic and had leukopenia. At
autopsy, the patients who died were found to have ulceration of Peyer patches. Which of the following infectious agents is most likely to produce these findings?
Salmonella typhi
Campylobacter jejuni
- Most common invasive bacterial enterocolitis in US.
- Mode of transmission
o Via contaminated poultry (undercooked chicken) or milk
- C/F
o Bloody diarrhea with crypt abscesses and ulcers
o Complication: GB syndrome
- Lab diagnosis
o Stool samples
 M/S - Exudate and streaks of blood
 Cultures grow the organism

Disorders of the Large Bowel


Hirschsprung disease
- Synonym
o Congenital aganglionic megacolon
- Patho-physiology
o Congenital absence of ganglion cells in rectum and sigmoid colon → intestinal obstruction. Migration
arrest of neural crest-derived cells before reaching anus leads to an aganglionic segment lacking both
Meissner and Auerbach plexus
- C/F
o In approximately 1 in 5000 to 8000 live births; M/F ratio of 4:1.
o Distention of the colon to greater than 6 or 7 cm in diameter
o Delayed passage of meconium
o Constipation, abdominal distension, and vomiting
o Associated with Down’s syndrome (2%) and Chaga’s disease
- Gross
o Affected segment is narrowed
o Proximal dilatation (megacolon)
- Microscopy
o Absence of ganglion cells in Auerbach myenteric & Meissener plexus
- Complications
o Enterocolitis, Toxic megacolon, Perforation
- Diagnosis: Rectal biopsy
- Treatment: Resection of affected segment
- A 24-year-old woman gives birth to term infant after an uncomplicated pregnancy. Apgar scores are 9 and 10 at 1 and 5 minutes after birth. The infant’s length
and weight are at the 55th percentile. There is no significant passage of meconium. Three days after birth, the infant vomits all oral feedings. On physical
examination, the infant is afebrile, but the abdomen is distended and tender, and bowel sounds are reduced. An abdominal ultrasound scan shows marked
colonic dilation above a narrow segment in the distal sigmoid region. A biopsy specimen from the narrowed region shows an absence of ganglion cells in the
muscle wall and submucosa. Which of the following is most likely to produce these findings? Hirschsprung disease
- A 2-year-old girl with a history of chronic constipation since birth is brought to the emergency room because of nausea and vomiting. Physical examination shows
marked abdominal distension. Abdominal radiography reveals distended bowel loops with a paucity of air in the rectum. A rectal biopsy shows an absence of
ganglion cells. Which of the following is the most likely diagnosis? Hirschsprung disease
Diverticulae
- Small herniations of colonic mucosa and submucosa through bowel wall
- Location
o At points of muscular weakness (anti- mesenteric border)
o Extend into epiploic appendages of serosal fat, especially in sigmoid colon
- Pathogenesis
o Multiple: Diverticulosis
o Acquired diverticula in the colon – diverticulosis, walls very thin, mainly mucosa and submucosa enclosed
within fat /intact peritoneal covering
o Prevalence : 50% in those older > 60 yrs
o Common in Western societies, in which the usual diet has less fiber with ↑ed constipation
o Sigmoid colon most common site in 95% cases
- C/F: Incidental finding
o Intermittent cramping, lower abdominal discomfort, constipation, distension, tenesmus,
Hematochezia : Massive loss of bright red blood per rectum
o Complications: ulcer, intermittent bleeding, diverticulitis. Rarely massive hemorrhage.
- Most common cause of hematochezia
o Diverticulosis not diverticulitis
o Scarring of vessels in diverticulitis prevents bleeding
- Complications
o Diverticulitis
 Impacted (fecolith) -ulceration/ischemia
 Diverticulitis -"left-sided appendicitis."
 CT scan /barium study
 ↑ed risk for perforation/abscess
 fistula.
 Peritonitis
 Stricture due to fibrosis
- A 65-year-old woman has a routine health maintenance examination. A stool sample is positive for occult blood. CT scan of the abdomen shows numerous air-
filled, 1-cm outpouchings of the sigmoid and descending colon. Which of the following complications is most likely to develop in this patient? Pericolic abscess
o Colonic diverticulosis may be accompanied by intermittent minimal bleeding and, rarely, by severe bleeding. One or more diverticula may become
inflamed (diverticulitis) or, less commonly, may perforate to produce an abscess, peritonitis, or both.
- A 74-year-old woman presents with 3 weeks of left lower quadrant abdominal pain, changes in bowel habits, and intermittent fever. Her temperature is 38°C
(101°F), respirations are 19 per minute, and blood pressure is 130/80 mm Hg. Physical examination shows left lower quadrant tenderness. A CBC reveals
neutrophilia. An abdominal-pelvic ultrasound examination is normal. Which of the following is the most likely diagnosis? Diverticulitis
Meckel's Diverticulum
- Congenital small bowel diverticulum
- 2 inches long, 2 feet from ileocaecal valve, 2% of population, 2 times more common in males
- Remnant of vitelline duct (omphalo-mesenteric duct) feces thru umblicus.
- True diverticulae composed of all layers of the normal small intestine
- O/E: Generally asymptomatic. Symptoms by 2 yrs.
o Intestinal bleeding, Melena, RLQ pain
o 50%- rests of pancreatic tissue or gastric mucosa
o Inflammation or perforation
o Peptic ulcer, occult bleeding
o Pain like Acute appendicitis / intestinal obstruction
- A 24-year-old man has developed abdominal pain and increasing fatigue over the past 6 months. On physical examination, he is afebrile and appears pale. On
palpation, there is mild pain in the right lower quadrant of the abdomen. There are no masses, and bowel sounds are active. Laboratory studies show
hemoglobin, 8.9 g/dL; hematocrit, 26.7%; MCV,74 μm3; platelet count, 255,000/mm3; and WBC count, 7780/mm3. His stool is positive for occult blood. Upper
gastrointestinal endoscopy and colonoscopy showed no lesions. One month later, he continues to experience the same abdominal pain. Which of the following is
most likely to cause this patient’s illness? Meckel diverticulum
Vascular Ectasia (Angiodysplasia)
- Prone to rupture and bleed into lumen Massive lower GI bleeding in elderly [Aged] > 6th decade
o Tortuous ectatic dilations of submucosal and mucosal Veins and venules. Enlarged & fragile vessels.
- Location
o Cecum and Ascending colon
- C/F
o Multiple episodes of rectal bleeding
o Colonoscopy / Angiography
o Rupture → GI bleeding &/or unexplained Anemia in elderly
- A 60-year-old man has had increasing fatigue for the past 8 months. On physical examination, he appears pale. On digital rectal examination, no masses are
palpable, but a stool sample is positive for occult blood. Auscultation of the abdomen shows active bowel sounds, and on palpation there areno masses or areas
of tenderness. Laboratory studies show hemoglobin, 8.3 g/dL; hematocrit, 24.6%; MCV, 73 μm3; platelet count, 226,000/mm3; and WBC count, 7640/mm3.
Colonoscopyshows no identifiable source of the bleeding. Angiography shows a 1-cm focus of dilated and tortuous vascular channels in the mucosa and
submucosa of the cecum. What is the most likely diagnosis? Angiodysplasia
Hemorrhoids
- Variceal dilations of the anal and perianal submucosal venous plexuses.
- Common after 50yrs.
- Predisposing conditions : straining at stool in the setting of chronic constipation and the venous stasis of pregnancy in younger women.
- May reflect portal hypertension, usually resulting from cirrhosis of the liver
- A 20-year-old woman in her ninth month of pregnancy has increasing pain on defecation and notices bright red blood on the toilet paper.
She has had no previous gastrointestinalproblems. After she gives birth, the rectal pain subsides, and there is no more bleeding. Which of
the following is the most likely cause of these findings? Hemorrhoids
Appendix
- Inflammation of appendix (appendicitis) -Affects approximately 10% of persons in USA and Western countries in their lifetimes
- Peak incidence in 2nd and 3rd decades
- Most common abdominal surgical emergency
- Pathogenesis in children
o (many have no demonstrable cause).
o Lymphoid hyperplasia (60% of cases) often secondary to a viral infection
o Examples- Measles virus, adenovirus or immunization
- Pathogenesis in adults
o Fecalith obstructs prox. lumen --↑ed intra-luminal pressure --- mucosal injury and
bacterial invasion.
o Commonest cause-pinworm infection (Enterobius vermicularis)
o Other pathogens
 E.coli
 Bacteroides fragilis.
- Clinical findings in sequence:
o Initial colicky peri-umbilical pain
 Irritation of unmyelinated afferent C fibers on visceral peritoneal surface - Refer pain to the midline
o Nausea, vomiting, fever
 Pain precedes nausea and vomiting
o Pain shifts to RLQ
 Rebound tenderness at McBurney's point (Blumberg's sign)
o Laboratory findings
 Neutrophilic leukocytosis with left shift
- A 20-year-old woman has had nausea and vague lower abdominal pain for the past 24 hours, but now the pain has become more severe. On physical
examination, the pain is worse in the right lower quadrant, and there is rebound tenderness. A stool sample is negative for occult blood. Abdominal plain film
radiographs show no free air. The result of a serum pregnancy test is negative. Which of the following laboratory findings is most useful to aid in the diagnosis of
this patient? Neutrophilia with left shift
- Epidemiology
o Adolescents and young adults
o No sex predilection
o Many are not classic! Acute Appendicitis mimics viral gastroenteritis, ruptured follicular cyst, ruptured
ectopic pregnancy, mesenteric lymphadenitis, and Meckel's diverticulitis
- Complications
o Peri-appendiceal abscess
 Most common; often associated with perforation
o Peritonitis
o Pyelophlebitis - Inflammation of portal vein
 Danger of portal vein thrombosis
 Radiograph shows gas in the portal vein
 Air is present in the portal vein on X-ray
o Septicemia
o Sub-phrenic abscess
- Diagnosis - Ultrasound, CT scan, gallium scan
- Treatment - Extraperitoneal drainage and antibiotics
- A 53-year-old woman has increasing abdominal girth for the past 2 years. On
physical examination she has abdominal distension. An abdominal CT scan shows Acute
multiple nodules on peritoneal surfaces along with low attenuation mucinous Appendicitis
ascites. Paracentesis is performed and cytologic examination of the fluid obtained
shows well-differentiated columnar cells with minimal nuclear atypia. Where did
this proliferative process most likely arise in this woman? Appendix
Acute Appendicitis-pathology
- Early: scant neutrophilic infiltrate, vascular congestion; dull red appendix
o Key criterion: neutrophils in muscular wall
- Later: prominent neutrophilic infiltrate, fibrinopurulent exudate, ulceration, necrosis
- Even later: extensive ulceration, necrosis, and gangrene
- Too late: Rupture and peritonitis

Disorders of the Large Bowel-2


Inflammatory bowel diseases
- Crohn’s disease (Regional enteritis)
- Ulcerative colitis
- Colitis of indeterminate type
- Epidemiology
o Females > males
o Caucasians > non-Caucasians
o Age distribution
 Teens and early 20s
o Increasing incidence
o UC- More common than CD
o HLA association
- Pathogenesis
o Strong immune responses against normal flora
 Abnormal CD4+ T cell responses
o Defects in epithelial barrier function
- Clinical Course
o Episodes of bloody diarrhea or stools with mucus
o Crampy lower abdominal pain
 CD ~ appendicitis
o Fever
o Peri-anal fistulas
o Extra-intestinal manifestations (UC>CD)
o Malabsorption (CD of small bowel)
- Diagnosis by exclusion
o Endoscopy and biopsy
Crohn’s disease
- Sites
o Any level of alimentary tract (mouth to anus). Most common siteTerminal ileum (80 %)
("terminal ileitis" / "regional enteritis" )
o May extend in both directions
o Uncommon site Rectum
- Pathology
o Acute disease
o Serosa and mesentery : creeping fat
o Edema and hyperemia
o “Skip lesions”
 Segmental involvement
 Mucosa- Patchy inflammation, ulceration alternating with normal regions
o NOD2 (nucleotide-binding oligomerization domain) gene mutations
 Th1 and Th17 reactions
o Transmural involvement
 Wall - Thickened with inflammation and fibrosis involving all layers of the wall and mesentery
 ~ a rubber hose
- Late stages
o Mucosa
 Cobblestone appearance with linear narrow ulcers-fissures . Elongated serpentine ulcers along long
axis of bowel
o Adhesions
 full thickness of wall involved
o Narrow lumen (string sign)
o Fissures and fistulae
- Gross: “Creeping fat appearance”
- Microscopy
o Mucosa
 Ulceration with nonspecific inflammation.
 Crypt abscess, distortion, bizarre branching, pseudopyloric metaplasia.
o Submucosa
 Lymphoid hyperplasia and fibrosis
o Submucosa/subserosa
 involved with secondary mucosal involvement.
o Non-caseating Granulomas
 Characteristic (only 35% cases)
o Sub-serosa
 Fibrosis, adhesions
- Complications
o Obstruction
 Terminal ileum
 Secondary to submucosal fibrosis, muscular hypertrophy stricture (Dilatation of bowel proximal
to stenotic involved segment)
o Fistula formation
 Secondary to perforation--involving adjacent loops of small bowel, colon, urinary bladder and vagina
o Abscess
 Mesenteric, peritoneal, and peri-rectal
o Malabsorption
 Secondary to mucosal disease & extensive surgical resection of small bowel
o Systemic complaints
 Arthritis (sacroiliitis, ankylosing spondylitis)
 Uveitis
 Erythema nodosum
 Migratory polyarthralgia
 Primary sclerosing cholangitis can happen but more common in UC.
o Risk of GIT carcinoma
 Slight, compared to ulcerative colitis
- Clinical features
o Intermittent attacks of mild diarrhea, fever, crampy abdominal pain, Fe deficiency, protein malabsorption
o Melena in 50% of cases with colon involvement
o Remissions and relapses
o Pain (in RLQ) ~ appendicitis
o Obstruction
o Associated disorders Presenting symptoms can be extra-luminal
- Radiography
o Lesions narrow the lumen --- obstruction
o Narrowing - Irregular and multiple (“String sign”)
- A 27-year-old man has sudden onset of marked abdominal pain. On physical examination, his abdomen is diffusely tender and
distended, and bowel sounds are absent. He undergoes surgery, and a 27-cm segment of terminal ileum with a firm,
erythematous serosal surface is removed. The microscopic appearance of a section through the excised ileum is shown in the
figure. Which of the following additional complications is the patient most likely to develop as a result of this disease process?
Enterocutaneous fistula
- A 26-year-old man has had intermittent cramping abdominal pain and low-volume diarrhea for 3 weeks. On physical examination,
he is afebrile; there is mild lower abdominal tenderness but no masses, and bowel sounds are present. A stool sample is positive for occult blood. The symptoms
subside within 1 week. Six months later, the abdominal pain recurs with perianal pain. On physical examination, there is now a perirectal fistula. Colonoscopy
shows many areas of mucosal edema and ulceration and some areas that appear normal. Microscopic examination of a biopsy specimen from an ulcerated area
shows a patchy acute and chronic inflammatory infiltrate, crypt abscesses, and noncaseating granulomas. Which of the following underlying disease processes
best explains these findings? Crohn disease
- A clinical study of adult patients with chronic bloody diarrhea is performed. One group of these patients is found to have a statistically increased likelihood for
the following: antibodies to Saccharomyces cerevisiae but not anti–neutrophil cytoplasmic autoantibodies, NOD2 gene polymorphisms, TH1 and TH17 immune
cell activation, vitamin K deficiency, megaloblastic anemia, and gallstones. Which of the following diseases is this group of patients most likely to have? Crohn
disease
- A 21-year-old man is brought to the emergency room with symptoms of acute intestinal obstruction. His
temperature is 38°C (101°F), respirations are 25 per minute, and blood pressure is 120/80 mm Hg. Physical
examination reveals a mass in the right lower abdominal quadrant. The patient subsequently undergoes surgery,
and a segmental lesion involving the terminal ileum is resected (shown in the image). Which of the following is the
most likely diagnosis? Crohn disease
- A 24-year-old man is brought to the emergency room with symptoms of acute intestinal obstruction. His
temperature is 38°C (101°F), respirations are 25 per minute, and blood pressure is 120/80 mm Hg. Physical
examination reveals a mass in the right lower abdominal quadrant. At laparoscopy, there are numerous small
bowel strictures and a fi stula extending into a loop of small bowel. Which of the following is the most likely
diagnosis? Crohn disease
Ulcerative Colitis
- Inflammatory disease of colon, limited to mucosa and submucosa (more mucosal involvement)
- Incidence: 7/100,000 population, more in whites, peak between 20 -25 years
- Familial association - 20% have affected relatives
- Begins in rectum and extends proximally in a continuous fashion
- Chronic, relapsing disease
o Attacks of bloody diarrhea with stringy mucous, cramps, lower abdominal pain, and fever
o Episodes preceded by stress
o Remissions – Spontaneous/steroid induced
- Sites of involvement
o Location
 Lt half of colon, rectum, Backwash ileitis., appendix
o Inflammation
 Only mucosa/submucosa
 Bowel wall NOT thickened
o Begins in rectum
 Extends to left colon
o No skip areas.
o “Pseudopolyps“-Inflammed bloody mucosa
 Mucosal ulcers extend laterally in submucosa; may interconnect/confluenced to create tunnels covered by "mucosal bridges".
o No linear fissures or fistulas.
- Microscopy
o Acute colitis
o Mucosal dysplasia
o Ulcers extend upto submucosa; not deeper
o Crypts
 Acute inflammation
 Ulcer
 Abscess
 Architecture Distortion
 Atrophy
o No granulomas/skip lesions
o No mural thickening, serosal surface is usually normal
o Higher risk of carcinoma development
 A 49-year-old woman has had abdominal cramps and diarrhea with six stools per day for the past month. She has a history of similar
episodes of self-limited pain and diarrhea, which have occurred multiple times during the past 20 years. Each episode lasts about 2 weeks
and resolves without treatment. Findings on physical examination are unremarkable, but a stool sample is positive for occult blood.
Laboratory studies show no ova or parasites in the stool. Colonoscopy shows diffuse and uninterrupted mucosal inflammation and
superficial ulceration extending from the rectum to the ascending colon. Colonic biopsy specimens from the area show the findings in the
figure. She is at greatest risk for developing which of the following complications? Adenocarcinoma
- Gland distortion, crypt abscess
- Main Symptoms of ulcerative colitis
o Colonic irritability
o Cramping
o Bloody diarrhea
o Excess mucous
o Hemorrhage
o Diffuse inflammation
- Radiology
o X-ray with barium enema
 Flattening of mucosal folds
 Pseudopolyps - Small filling defects
- Complications
o Colonic carcinoma
 Carcinomas are frequently preceded by dysplasia
 Patients with chronic colitis should be examined every 6-12 months by colonoscopy/biopsy to detect dysplasia
o Toxic megacolon
 Acute dilatation of colon due to loss of muscle tone, with associated systemic toxicity and shock
o Systemic complications of UC
o Arthritis of large joints (knees and ankles) in lower extremities
 Migratory polyarthritis, sacroilitis
o Skin manifestations
 Erythema nodosum
 Multiple small, raised, painful red lumps over legs
 Pyoderma
 Pyogenic (i.e. pus-forming) infection of skin.
 Pyoderma gangrenosum - Chronic eruption of spreading, undermined ulcers with central healing
o Uveitis
o Sclerosing Cholangitis
 Intra and/or extra-hepatic bile ducts
 Inflammation and fibrosis ---- obstructive jaundice
 Look for underlying IBD
 Cholangiograms
 Segmental, beaded involvement of large bile ducts
- A 35-year-old woman has had increasing lower back pain for 5 years. During the past year she also has had arthritic pain involving the knees, hips, and wrists. A
stool sample is positive for occult blood. A pelvic radiograph shows changes consistent with sacroiliitis. A colonoscopy is performed, and she undergoes a total
colectomy. The figure shows the gross appearance of the colectomy specimen. What is the most likely underlying mechanism of the illustrated condition?
Development of TH17 immune responses
- A 27-year-old woman presents with a 9-month history of bloody diarrhea and crampy abdominal pain. Three weeksago, she noticed that her left knee was
swollen, red, and painful. Her temperature is 38°C (101°F), respirations are 32 per minute, and blood pressure is 130/90 mm Hg. Abdominal palpation reveals
tenderness over the left lower quadrant. Laboratory studies show moderate anemia, with a hemoglobin level of
9.3 g/dL. Microscopic examination of the stool reveals numerous red and white blood cells. A diffusely red,
bleeding, friable colonic mucosa is visualized by colonoscopy. The colon is subsequently removed and the surgical
specimen is shown in the image. Which of the following is the most likely diagnosis? Ulcerative colitis
- A 44-year-old woman complains of having yellow eyes, dark urine, and recurrent fever for about 3 months. She has
a long history of chronic diarrhea. On physical examination, the patient is thin and jaundiced. The liver edge
descends 1 cm below the right costal margin and is nontender. Laboratory studies show elevated serum bilirubin of
3.8 mg/dL, normal levels of AST and ALT, and an elevated level of alkaline phosphatase (440 U/dL). Endoscopic
retrograde cholangiopancreatography demonstrates a beaded appearance of the extrahepatic biliary tree. Which
of the following is the most likely underlying cause of diarrhea in this patient? Ulcerative colitis
- A 25-year-old woman is brought to the emergency room with symptoms of acute intestinal obstruction. The
patient has an 8-month history of blood-tinged diarrhea and cramping abdominal pain. Her temperature is 38°C
(101°F), and respirations are 32 per minute. There is abdominal tenderness to palpation. Laboratory studies show
moderate anemia, with serum hemoglobin of 9.3 g/dL. Microscopic examination of the stool reveals numerous RBCs and WBCs. A CT scan of the abdomen shows
massive distention of the transverse colon. Which of the following is the most likely underlying cause of this patient’s colonic disorder? Ulcerative colitis
- A 25-year-old woman presents with persistent bloody diarrhea of 4 weeks’ duration. She has experienced severe abdominal cramping for the past 3 days. Her
temperature is 38°C (101°F), respirations are 22 per minute, and blood pressure is 120/70 mm Hg. Physical examination reveals abdominal tenderness and mild
abdominal distension. Bowel sounds are diminished. Laboratory studies show mild hypochromic, normocytic anemia. Stool cultures are negative for pathogens,
and no ova or parasites are detected. A blood test for Clostridium diffi cile toxin is negative. Recto-sigmoidoscopy shows hemorrhagic mucosal lesions in the
distal colorectal region. A biopsy of the colon reveals crypt abscesses, basal lymphoplasmacytosis and crypt distortion. Which of the following represents the
most common extraintestinal manifestation of the colonic disorder in this patient? Arthritis
Indeterminate colitis
- In 10% cases there is clinical and pathologic overlap between Cohn disease and Ulcerative colitis and definite diagnosis is not possible
- * Serology can be helpful
o p-ANCA positive in 75% of ulcerative colitis
o Antibodies to Saccharomyces cervisiae present in Crohn disease
- A 30-year-old woman has suffered intermittent bouts of lower abdominal pain and low-volume diarrhea for the past 2 years. On colonoscopy there is friable
mucosa from the rectum to the ascending colon, and a perianal fistula is noted. Biopsies are taken and on microscopic examination show acute and chronic
mucosal inflammation with focal erosion. Her stool is negative for ova, parasites, and bacterial pathogens. Which of the following ongoing testing procedures is
most useful for long-term follow-up of this woman? Biopsy screening for dysplasia
Disorders of the Large Bowel-3
- Non-neoplastic Polyps
o Hyperplastic polyps Cause: decreased epithelial cell turnover
o Hamartomatous polyps & delayed shedding  piling up of Goblet cells
o Juvenile polyps
o Peutz-Jeghers polyps
o Inflammatory polyps
- Neoplastic Epithelial Lesions
o Benign polyps
o Adenomas
o Malignant lesions
o Adenocarcinoma
- Other Tumors
o Gastrointestinal stromal tumors
o Carcinoid tumor
o Lymphoma
Hyperplastic (Non-neoplastic) polyp
- Most common type in adults
o In 50% persons >60 years
- Location
o Sigmoid colon Non-
o Size < 5mm. Smooth, nodular. neoplastic
- No malignant potential
- Cause: decreased epithelial cell turnover & delayed shedding  piling up of Goblet cells polyp
- M/S- “Saw-tooth" / serrated appearance
o Well-formed glands with crowding
o Goblet cells and absorptive cells with exaggerated crypt architecture
o No nuclear atypia
- A 55-year-old man undergoes routine colonoscopy. A small, raised, mucosal nodule measuring 0.4 cm in diameter is identified in the rectum and resected. The
surgical specimen is shown in the image. Microscopic examination reveals goblet cells and absorptive cells with exaggerated crypt architecture, but no signs of
nuclear atypia. Which of the following is the most likely diagnosis? Hyperplastic polyp
Juvenile polyps [common in < 5 yrs]
- Located in rectum
o Prolapse out of rectum ---bleeding
- Solitary polyp Juvenile
o Hamartomatous Smooth surface with enlarged cystic spaces on cut section polyps
- Juvenile polyposis
o Autosomal dominant or non-hereditary 3-100 polyps
- The mother of a 4-year-old child notes blood when laundering his underwear. Physical examination reveals a rectal mass. On
proctoscopy, there is a smooth-surfaced, pedunculated, 1.5-cm polyp. It is excised and microscopically shows cystically dilated
crypts filled with mucin and inflammatory debris, but no dysplasia. What is the most likely diagnosis? Juvenile polyp
Inflammatory Polyp
- Solitary rectal ulcer syndrome
o Rectal bleeding, mucus discharge and inflammatory lesion of anterior rectal wall
o Impaired relaxation of anorectal sphincter
o Chronic inflammatory changes
o Other hamartomatous polyps
 Cronkhite-Canada syndrome:
 Nonhereditary
 Polyps + ectodermal abnormalities of nails, hair & skin
 Cowden syndrome:
 Multiple hamartomas
 GI polyps with increased risk of other cancers
 Germ-line mutations in PTEN gene

Peutz-Jeghers syndrome
- Mutation of the STK11/LKB1 tumor suppressor gene in most cases located on band 19p13.3.
o Multiple hamartomatous polyps (primarily in small intestine)
- Melanin pigmentation of oral mucosa
o Median age 11 year
o Dark blue to brown macules around the mouth, eyes, nostrils, buccal mucosa,
palmar surface of hands, genitalia & perianal region.
- Non-neoplastic hamartomatous polyp in small intestine. Rare in colon & stomach
- Autosomal dominant
- Increased risk of cancer
o Lung, pancreas, breast, uterus
- Gross: Pedunculated large polyps, lobulated contour
- Microscopy: Arborizing network of connective tissue, smooth muscle, lamina propria & glands
- A 5-year-old girl is brought to the physician after her parents noticed red blood in her stool. Physical examination reveals
mucocutaneous pigmentation. Small bowel radiography discloses multiple, small- to medium-sized polyps that are
diagnosed pathologically as hamartomas. Which of the followingis the most likely diagnosis? Peutz-Jeghers polyp
Adenomatous colonic polyps
- Definition
o Benign neoplasm of colonic mucosa
o All adenomas arise from epithelial proliferation and dysplasia and so
are neoplastic
o Potential to progress to
adenocarcinoma
- Clinical findings:
o Asymptomatic
o Occult bleeding --- iron
deficiency anemia
- Diagnosis
o Hemoccult +ve stools
o Endoscopy
- Pathology
o - Small or large
 Pedunculated tubular (60%) / sessile villous (10%) / Tubulo-villous (30%)
 Velvet or raspberry surface
- A 53-year-old woman undergoes a routine checkup. The only abnormal finding is a stool specimen that contains occult blood.
Colonoscopy shows a 1.5-cm, solitary, rounded, erythematous polyp on a 0.5-cm stalk at the splenic flexure. The polyp is
removed; its histologic appearance is shown in the figure at low (A) and high (B) magnifications. Her colonic lesion is most
likely associated with which of the following? Low risk for development of carcinoma

Tubular adenomas
- Tubular Adenoma- 50% in rectosigmoid , single.
- Size : from 0.3 to 2.5 cms.
- The stalk is covered by normal colonic mucosa, but head is composed of neoplastic
epithelium
- Microscopic examination shows irregular crypts lined by pseudostratified epithelium with
hyperchromatic nuclei.
- Clinical Findings:
o Asymptomatic
o Large polyps can bleed  Fe deficiency anemia
- A 65-year-old woman undergoes routine colonoscopy. During the procedure, a 2-cm mass is identified in the rectosigmoid region and
resected. The surgical specimen is shown in the image. Microscopic examination shows irregular crypts lined by pseudostratified
epithelium with hyperchromatic nuclei, without dysplastic features. Which of the following is the most likely diagnosis for this patient’s
colonic lesion? Tubular adenoma
Villous adenomas
- Older persons, mostly in rectum and rectosigmoid
- Generally sessile, up to 10 cm in diameter
- Frond-like villiform extensions of mucosa covered by dysplastic,
disorderly columnar epithelium
- Overt or occult rectal bleeding, hypoproteinemia, hypokalemia
- Villous adenoma: higher risk of developing colon cancer (40%)
- Risk factors for malignancy in adenomas
o Size
 Most imp.factor
 Cancer- Rare in tubular adenomas < 1 cm
 >4cm (40% risk of malignancy)
o Pathology
 Tubular/Villous Pedunculated/ Sessile
 Sessile villous adenomas> 4 cm (30-40% risk)
o Degree of dysplasia
 Severe dysplasia --- more likely to progress
- A 70-year-old man has a routine health maintenance examination. On physical examination, there are no remarkable findings, but a stool sample is positive for
occult blood. A colonoscopy is performed and shows a 5-cm sessile mass in the upper portion of the descending colon
at 50 cm from the anal verge. The histologic appearance at low power of a biopsy specimen of the lesion is shown in
the figure. The patient refused further workup and treatment. Five years later, he has constipation, microcytic anemia,
and a 5-kg weight loss over 6 months. On surgical exploration, there is a 7-cm mass encircling the descending colon.
Which of the following neoplasms is he now most likely to have? Adenocarcinoma
- A 63-year-old woman complains of rectal bleeding of 1 week in duration. Laboratory studies show hypochromic,
microcytic anemia (hemoglobin = 7.6 g/dL and MCV = 70 μm3). Colonoscopy reveals a large polypoid mass, which is
removed (surgical specimen shown in the image). The arrow points to a malignant tumor. The patient asks about the
relative risk of cancer arising in various types of gastrointestinal polyps. Which of the following types of colonic polyps
is most likely to undergo malignant transformation? Villous adenoma

Familial adenomatous polyposis (FAP)


- Synonym
o Adenomatous polyposis coli (APC)
- Genetics
o Autosomal Dominant- Inactivation of APC- suppressor gene chr. 5q21
- Pathology
o All patients develop tubular adenomas and cancer
o Polyps mostly tubular adenomas begin to develop b/w 10-20 yrs of age.
- Complications
o Malig. transformation before 30 years of age
o Prophylactic colectomy is recommended.
o By age 40, virtually 100% invasive adenocarcinoma
- Diagnosis: >100 adenomatous polyps on endoscopy
- A 19-year-old man is advised to see his physician because genetic screening has detected a disease in other family members. On
physical examination, a stool sample is positive for occult blood. A colonoscopy is performed, followed by a colectomy. The
figure shows the gross appearance of the mucosal surface of the colectomy specimen. Microscopic examination shows these
lesions are tubular adenomas. Molecular analysis of this patient’s normal fibroblasts is most likely to show a mutation in which
of the following genes? APC
- A portion of the large bowel was removed from a 34-year-old man with a familial disease that affects his gastrointestinal tract.
The surgical specimen is shown in the image. This patient most likely carries a germline mutation in which of the following
genes? APC
Gardner Syndrome
- Autosomal dominant
- Variant of FAP characterized by
o Numerous colonic adenomatous polyps
o Multiple osteomas – mandible, skull, long bones
o Fibromatosis (desmoid tumors)
o Epidermal inclusion cysts
o Abnormal dentition (unerupted / supernumerary teeth)
o Increased incidence of duodenal and thyroid cancers
Turcot Syndrome
- Rare variant of FAP characterized by
o Autosomal recessive polyposis syndrome
o Associated with CNS tumors
 Astrocytomas, Medulloblastomas, Glioblastoma
Hereditary nonpolyposis colorectal cancer (HNPCC)
- Synonym
o Lynch syndrome
- Genetics
o Autosomal dominant
o Mutation of DNA nucleotide mismatch repair gene (MLH1, MSH-2, MSH-6) ***
- Colon cancer
- Increased risk of endometrial and ovarian cancer, gastric Ca, skin.
- A 33-year-old man has a routine health maintenance examination. A stool sample is positive for occult blood.
On colonoscopy, a 6-cm ulcerative lesion is seen projecting into the cecum. There are three smaller sessile
lesions from 1 to 3 cm in size. The microscopic appearance of a section of the ulcerated lesion is shown in the
figure. The smaller lesions are reported as sessile serrated adenomas. Which of the following molecular
biological events is thought to be most critical in the development of such lesions? Defective DNA mismatch
repair gene
- A 44-year-old woman has had increasing abdominal distention for the past 6 weeks. On physical examination, there is an abdominal fluid wave, and bowel
sounds are present. Paracentesis yields 1000 mL of slightly cloudy serous fluid. Cytologic examination of the fluid shows malignant cells consistent with
adenocarcinoma. Molecular analysis of these cells shows an MSH2 gene mutation with microsatellite instability. Her medical history indicates that she has had no
major medical illnesses and no surgical procedures. Her sister was diagnosed with endometrial cancer and her brother had carcinoma of the stomach. Which of
the following conditions is the most likely cause of this patient’s symptoms? Lynch syndrome
Colorectal Carcinoma
- Colorectal cancer ranks second only to bronchogenic carcinoma among cancer killers in the United States
- 98% - adenocarcinomas
- Peak incidence : 60 to 70 years, males 20% more than females
- In a young person: preexisting ulcerative colitis / polyposis syndromes must be suspected
Colonic adenocarcinoma
- 2nd most common tumor in terms of incidence and mortality in US
- Risk factors
o Low-fiber diet
o Diet low in fruits and vegetables
o High red meat and animal fat consumption
o Decreased intake of protective micronutrients like vitamins A, C, and E
o Several recent epidemiologic studies suggest that use of aspirin and other NSAIDs exerts a protective effect
against colon
o Adenomatous polyps
o Hereditary polyposis
syndromes
o Lynch syndrome (HNPCC)
o Ulcerative colitis (IBD)
Adenoma-Carcinoma sequence
- Locations for colon cancer
o Recto-sigmoid (50%)
o Ascending colon (15%)
o Descending colon (15%)
o Transverse colon and
cecum (10% each)
- Screening tests for colon cancer
o Fecal occult blood test
 Not very sensitive or specific for colon cancer
o Colonoscopy (gold standard) with biopsy
- Left-sided cancers tend to obstruct
o Bowel diameter < right colon
o Annular, encircling ("napkin-ring“) lesions
o Change in bowel habits
o Constipation/diarrhea with/without bleeding
- Right-sided cancers tend to bleed.
o Bowel diameter > left colon
o Polypoid, exophytic tumors
o Blood in stool and iron deficiency
- Pattern of spread
o Lymphatic spread to mesenteric lymph nodes
o Distant spread to liver, lungs, and bone
- Detection and diagnosis
Left-sided cancers Right-sided cancers
o Digital rectal examination/fecal testing for occult blood
o Barium enema, sigmoidoscopy, colonoscopy with biopsy
o CT / other radiographic studies to assess metastatic spread
o Molecular detection of APC mutations in epithelial cells, isolated from stools
o Detection of abnormal patterns of methylation in DNA isolated from stool cells
- Colorectal carcinoma - staging
o The single most important prognostic indicator of colorectal carcinoma is the extent (stage) of the tumor at the time of diagnosis
o NO NEED TO MEMORIZE THIS STAGING
- Management
o Surgical resection
o Chemotherapy for metastatic disease
o Monitor CEA levels
 Elevated CEA levels may be seen in
 CA lung, breast, ovary, urinary bladder, and prostate
 Non-neoplastic disorders
o Alcoholic cirrhosis, pancreatitis, ulcerative colitis
o Use
 To assess possible recurrence following resection of a primary tumor.
 If total removal has been accomplished, CEA disappears.
- A 73-year-old man has noted a change in bowel habits for the past year. Defecation is more difficult and the caliber of stools has
decreased. On physical exam, there are no abnormal findings except for stool positive for occult blood. Colonoscopy is performed
for the first time in this man, followed by colonic resection with the gross appearance shown in the figure. Which of the following
molecular abnormalities has most likely led to these findings? Mutation in a DNA mismatch repair gene
o The figure shows an encircling mass that is typical of adenocarcinoma of the descending colon. Such cancers likely to
obstruct, but they can also bleed a small amount over months to years, causing iron deficiency anemia. The APC gene,
a negative regulator of β-catenin in the WNT signaling pathway, is associated with familial adenomatous polyposis
syndrome and most sporadic colon cancers, as in this case. This pathway also is known as the adenoma-carcinoma
sequence because the carcinomas develop through an identifiable series of molecular and morphologic steps.
- A 59-year-old man complains of progressive weakness. His friends have noticed that he has become pale, and he reports that his
stools are tinged with blood. On abdominal palpation, there is fullness in the right lower quadrant. Laboratory studies show iron-
deficiency anemia, with a hemoglobin level of 7.4 g/dL. Stool specimens are positive for occult blood. Colonoscopy reveals an elevated and centrally ulcerated
lesion of the sigmoid colon. The biopsy is shown in the image. Which of the following is the most likely diagnosis? Adenocarcinoma
Carcinoid tumors
- Neuro-endocrine tumor (NET) often producing serotonin (pt may have flushing)
- Location
o Appendix (most common); Terminal ileum, rectum
- Metastasis to liver may→ carcinoid syndrome.
- Most carcinoids contain chromogranin A, synaptophysin, and neuron-specific
enolase
- Pathology
o Solitary or multicentric firm, yellow-tan nodules.
o Sub-mucosal masses, ulceration.
o Nests of cells with round and monotonous, speckled nuclei and abundant pink cytoplasm (salt & pepper
appearance).
o Cytoplasmic secretory dense-core granules-EM.
- Diagnosis
o Urinary 5-HIAA (5-hydroxy indole acetic acid)
- A 26-year-old man is brought to the emergency department after sustaining abdominal gunshot injuries. At laparotomy,
while repairing the small intestine, the surgeon notices a 1-cm mass at the tip of the appendix. The yellow-tan
submucosal mass is removed, and the microscopic appearance of the mass is shown in the figure. Immunohistochemical
staining is positive for chromogranin and synaptophysin but negative for Ki-67. Which of the following is the most likely
cell of origin of this lesion? Neuroendocrine cell
- A 55-year-old man experiences episodes of diaphoresis, dyspnea, and diarrhea for 10 months. On physical examination he has midabdominal discomfort with
deep palpation, and bowel sounds are reduced. There are no abnormal findings with upper endoscopy. Abdominal CT scan shows three nodules in the liver, from
1 to 3 cm in size. Laboratory studies show a high level of serum 5-hydroxyindoleacetic acid (5-HIAA). Camera endoscopy is performed, and on review of the
images, there is a midjejunal mass that partially obstructs the lumen. At laparotomy a 5-cm submucosal jejunal mass is resected, and on microscopy it is
composed of nests and trabeculae of round cells with pink, granular cytoplasm. The cells of this mass are most likely related to which of the following
embryologic derivatives? Neural crest
Disorders of the Liver and Biliary Tract 1 & 2
Clinical features of Liver disease
- Bilirubin metabolism
Laboratory Evaluation of Liver Disease- I
- (I).Bilirubin Excretion
o 1. CB <20% Unconjugated hyperbilirubinemia: e.g., extravascular
hemolytic anemias eg.hereditary spherocytosis
o 2. CB 20%–50% Mixed hyperbilirubinemia (e.g., viral hepatitis)
o 3. CB >50% Conjugated hyperbilirubinemia (e.g., liver cholestasis)
o 4. Urine bilirubin Bilirubinuria: viral hepatitis, intrahepatic or
extrahepatic obstruction of bile ducts
o 5. Urine UBG Increased urine UBG: extravascular hemolytic anemias,
viral hepatitis
o 6. Absent urine UBG: liver cholestasis
Laboratory Evaluation of Liver Disease-II
- (II)Liver Cell Necrosis –
o 1. Serum alanine transaminase (ALT) Present in the cytosol
 Specific enzyme for liver cell necrosis
 ALT > AST: viral hepatitis
o 2. Serum aspartate transaminase (AST) Present in mitochondria
 Alcohol damages mitochondria:
 AST > ALT indicates alcoholic hepatitis.
- (III) Cholestasis GGT & ALP are synthesized by bile duct epithelium.
o 1. Serum γ-glutamyltransferase (GGT)
 Intrahepatic or extrahepatic obstruction to bile flow
 Induction of cytochrome P450 system (e.g., alcohol): increases GGT
o 2. Serum alkaline phosphatase (ALP)
 Increased GGT and ALP: liver cholestasis.
 Normal GGT and increased ALP: source of ALP other than liver (e.g.,osteoblastic
activity in bone)
- Clinical features of Hyperbilirubinemia
o Obstructive/ post hepatic jaundice
o Conjugated bilirubinuria
o Pale stools
o Itching in obstructive jaunduce
Laboratory Evaluation of Liver Disease-III
- (IV)Hepatocyte Function
o 1. Serum albumin Albumin is synthesized by the liver
 Hypoalbuminemia: severe liver disease (e.g., cirrhosis)
o 2. Most of coagulation factors are synthesized in the liver – PT & PTT Increased in
severe liver disease
o 3. Factor V Decreased in severe liver disease
o 4. Blood urea nitrogen (BUN) Urea cycle is present in the liver.
 Decreased serum BUN: cirrhosis, fulminant hepatitis
o 5. Serum ammonia Ammonia is metabolized in the urea cycle in the liver.
 Derives from large bowel and amino acid degradation
 Increased serum ammonia: cirrhosis, Reye syndrome
Laboratory Evaluation of Liver Disease-IV
- (V) Immune Function
o Serum IgM Increased in primary biliary cirrhosis
o Antimitochondrial antibody Primary biliary cirrhosis
o Anti–smooth muscle antibody Autoimmune hepatitis
o Antinuclear antibody (ANA) Autoimmune hepatitis
- (VI)Tumor Marker : α-Fetoprotein (AFP) Hepatocellular carcinoma
o A 68-year-old man complains of vague abdominal pain, intermittent fever, and a 20-lb (9-kg) weight
loss over the past 6 months. For the past 12 years, he has suffered from chronic hepatitis B. On
physical examination, the patient shows diffuse abdominal tenderness, hepatomegaly, and mild
jaundice. A CT scan of the abdomen reveals a diffusely nodular liver, with a dominant mass
measuring 3 cm in diameter. A needle biopsy is shown in the image. Which of the following serum
markers is useful for monitoring the progression of disease in this patient? Alpha-fetoprotein
o
Lab findings in selected liver disease
Jaundice – Causes
- Pre-hepatic jaundice - CB <20% Unconjugated hyperbilirubinemia Increased
production of bilirubin – Hemolytic anemia
- Hepatic Jaundice – CB 20%–50% Mixed hyperbilirubinemia
Decreased uptake; decreased conjugation or decreased hepatocytic
excretion, intrahepat6ic biliary obstruction
- Post hepatic Jaundice –CB >50% Conjugated hyperbilirubinemia
cholesteric Jaundice – Bile duct obstruction by stones, stricture, tumor
Lab findings in different types of Jaundice
- Neonatal jaundice
1. Physiologic Jaundice
o 50-60 % of all newborns are jaundiced in the first week of life.
o Total serum bilirubin peaks at age 3–5 d and 7th day in preterm.
o Mean peak total serum bilirubin is 6 mg/dL (higher in pretrerm
infants).
o Because of ↑ risk of bilibubin encephalopathy
“physiologic” jaundice is more difficult to define and jaundice
should be followed closely.
- Neonatal Jaundice
2. Non-physiologic/pathological jaundice:
o Jaundice in the first 24 hours
o Bilirubin rising faster than 5 mg/dL in 24 hours
o Clinical jaundice >1 week
o Direct bilirubin >2 mg/dL
o In healthy term infants total serum bilirubin concentration >15 mg/dL
o Lower levels in preterm infants, “sick” infants, and hemolytic disease
Bilirubin encephalopathy:
- The mildest form - sensorineural hearing loss due to damage to the cochlear nuclei.
- Severe encephalopathy causes kernicterus – cerebral palsy, developmental delay, Hearing defect
Factors predisposing to neurotoxicity of unconjugated hyperbilirubinemia include:
o When bilirubin concentration exceeds the binding capacity of serum albumin
o Displacement of bilirubin from albumin by acidosis or certain drugs (e.g., sulfonamides, ceftriaxone)
o Sepsis
o Preterm infants due to↑ risk due lower serum albumin concentrations and ↑ risk for
o acidosis and sepsis.
- Phototherapy for unconjugated hyperbilirubinemia in Newborn
- Causes of unconjugated (indirect) hyperbilirubinemia:
o 1. Increased lysis of RBCs ( hemolytic anemia)
 a. Isoimmunization (blood group incompatibility: Rh, ABO and minor blood groups)
o b. RBC enzyme defects (e.g., G6PD deficiency, pyruvate kinase deficiency)
o c. RBC structural abnormalities (hereditary spherocytosis, elliptocytosis)
 Infection (sepsis, urinary tract infections
o d. Polycythemia – Infants born to Diabetic mothers
- 2. Unconjugated hyperbilirubinemia due to Decreased hepatic uptake and conjugation of bilirubin
o Inherited conjugation defect (Non-hemolytic Unconjugated Hyperbilirubinemia):
o a. Gilbert Syndrome
o b. Crigler Najjar Syndrome (very rare)
o Breastmilk Jaundice (pregnanediol inhibits glucuronyl transferase activity)
Gilbert Syndrome
- Affects 5-6% population
- Common benign inherited AD disorder
- Unconjugated hyperbilirubinemia
- Jaundice related to stress (fasting, infection, fever, alcohol etc.)
- Mechanism : Bilirubin glucuronosyl-transferase (UGT1A1) deficiency
- No Clinical consequence
Crigler-Najjar Syndrome
- Unconjugated hyperbilirubinemia
- Mechanism : Bilirubin glucuronosyl-transferase (UGT1A1) deficiency
- Type I: Total absence of UGT1A1
AR-fatal because of kernicterus
- Type II: AD; non-fatal; jaundice
- Less severe UGT1A1 deficiency
Dubin- Johnson Syndrome
- Autosomal Recessive
- Conjugated hyperbilirubinemia
- Decreased bilirubin excretion
- Absent canalicular transport protein (MRP2)
- Brown/Black pigmentation of liver
- Asymptomatic, but chronic jaundice
Rotor syndrome
- Autosomal recessive
- Conjugated hyperbilirubinemia
- Multiple defects in hepatocellular uptake & excretion of bilirubin
- Similar to Dubin-Johnson but without liver pigmentation
- No clinical consequences
Cholestasis
- Hepatocellular dysfunction OR
- Biliary obstruction
Cholestasis: Cellular Effects
- Intra-cytoplasmic bile pigments
- Dilated canaliculi
- Apoptotic single hepatocytes
- Kupffer cells contain bile pigments
- Bile ductular proliferation
- Dilated bile ducts
- Degeneration of limiting plates
Obstructive (Cholestatic) Liver Disease
- 1. Intrahepatic cholestasis - blockage of the intrahepatic bile ducts
o Causes :
 `(1) Drugs (e.g., oral contraceptive pills [OCPs], anabolic steroids) -Drugs most common
cause of intrahepatic cholestasis
 (2) Neonatal hepatitis
 (3) Pregnancy-induced cholestasis (estrogen)
- 2. Extrahepatic cholestasis
o Blockage of common bile duct (CBD)
o Causes
 (1) Stone usually originating from the gallbladder (most common cause)
 (2) Primary sclerosing cholangitis
 (3) Primary biliary atresia
 (4) Carcinoma at head of pancreas
 (5) Parasites - Liver flukes- Clonorchis sinensis
- Gross and microscopic findings in cholestatic liver disease
o Greenish Liver
o Dilated Bile ductules filled with yellowish green bile
Bile lakes and necrosis also seen
- Clinical presentation
o Jaundice and icterus
o Pruritis -due to increased plasma levels of bile acids
o Skin xanthomas-focal accumulations of cholesterol due to hyperlipidemia and impaired excretion of cholesterol.
o Abdominal pain, fever, and chills
o Dark urine (bilirubinuria)
o Pale clay-colored stools
- Lab investigations
o Elevated conjugated bilirubin
o Elevated serum alkaline phosphatase
o Elevated γ-glutamyl transpeptidase
o Elevated 5’-nucleotidase
Primary Biliary Atresia
- Complete/partial obstruction of the lumen of
the extrahepatic biliary tree within the
first 3 months of life
- Accounts for half of children liver transplantation
o 1. Perinatal form (more common)
 Normally developed biliary tree destroyed after birth
 Etiology remains unknown (? viral infection and autoimmune reactions)
o 2. Fetal form (20% of cases)
 Intrauterine developmental anomaly of extrahepatic biliary tree
 Associated with - malrotation of abdominal viscera, situs inversus, and congenital heart
disease)
- Inflammation and fibrosing stricture of the hepatic or common bile ducts
o Labs:
1. serum bilirubin 6 to 12 mg/dL,
2. moderately elevated aminotransferase and alkaline phosphatase levels
- Complications & Rx:
Cirrhosis develops within 3 to 6 months of birth
- surgical correction for Type 1 & 2
- With cirrhosis – Liver transplantation

Primary sclerosing cholangitis


- Chronic, progressive cholestatic liver disease
- Characterized by Inflammation, obliterative fibrosis and segmental dilation of intra-hepatic & extra-hepatic
bile ducts
- Cause - unknown
- Epidemiology
o Middle-aged males
o Associated with ulcerative colitis
- Complications - Biliary cirrhosis; Cholangiocarcinoma
- Inv : Endoscopic retrograde cholangiopancreatography:
- “Beaded appearance” of the extrahepatic biliary tree (arrows)
Liver Failure – Acute/ Fulminant
- Definition—acute liver failure with encephalopathy and coagulopathy within 26 weeks of initial liver injury
without preexisting liver condition
- Causes:
o (1) Drugs -e.g., acetaminophen most common cause in US-50%
o (2) Viral hepatitis (second most common cause) - HAV,HBV
o (3) Reye syndrome, Wilson disease, autoimmune hepatitis
- Gross and microscopic findings
o Liver is small , bile stained, soft
o Areas of necrosis with acute inflammatory cells surrounded by normal hepatocytes
- Laboratory findings
o Decrease in transaminases
 Liver parenchyma is destroyed.
- Morphologic alterations causing liver failure
o Massive hepatic necrosis
 Fulminant viral hepatitis
 Drugs & chemicals- Acetaminophen, halothane, mushroom poisoning
o Chronic liver disease
 Chronic hepatitis ---- cirrhosis
o Hepatic dysfunction without overt necrosis
 Tetracycline toxicity

Cirrhosis
- Definition: Irreversible diffuse fibrosis of the liver with formation of regenerative nodules
o End-stage liver disease
o Disruption of entire liver architecture
o Complication:
1. Portal hypertension
2. Chronic Liver failure
3. Hepatocellular carcinoma
- Etiology
o Alcoholic liver disease - 60-70%
o Viral hepatitis-B,C,D - 10%
o Biliary diseases - 5-10%
o Primary hemochromatosis- 5%
o Wilson disease - Rare
o α1-Antitrypsin deficiency - Rare
o Cryptogenic cirrhosis - 10-15%
- Pathogenesis of cirrhosis
- Chronic hepatocyte injury chronic inflammation cirrhosis of the liver
- Gross
o Micronodular (Nodules < 3mm)
o Macronodular (Nodules > 3mm)
o Mixed micronodular and macronodular- End-stage
- Microscopy
o H&E section: Fibrosis with chronic inflammatory cells surrounding
regenerating nodules ; Note the loss of normal liver architecture
o Massons Trichrome stain: Highling the collagen ( in blue)
surrounding the nodules of hepatocyte
- Portal hypertension (increased resistance to portal blood flow)
o Causes
 1.Pre-hepatic
 Portal vein thrombosis
 Massive splenomegaly
 2.Intra-hepatic
 Cirrhosis
 Schistosomiasis, massive fatty change
 Post-hepatic
o Severe right-sided heart failure, constrictive pericarditis, Budd-Chiari syndrome(abdominal pain, ascites & hepatomegaly)
- Portal hypertension secondary to cirrhosis
- Consequences of portal HT
o Ascites
o Congestive splenomegaly
o Port-systemic venous shunts
 Esophageal varices
 Hemorrhoids
 Caput medusae
 dilated veins on abdomen in cirrhosis
o Hepatic encephalopathy
- Clinical manifestations of Cirrhosis
o Decreased detoxification
 Hepatic encephalopathy
 Spider angioma
 Palmar erythema
 Gynecomastia
o Decreased synthesis
 Hypo-albuminemia
 Decreased clotting factors
o Hepatorenal syndrome
- Hepato-renal syndrome
o Renal failure without renal parenchymal disease
o Due to decreased renal blood flow
o Preservation of renal tubular function
- Hyper-estrinism in males
o Pathogenesis
 Liver cannot degrade estrogen and 17-ketosteroids (e.g., androstenedione).
 Androstenedione is aromatized into estrogen in the adipose cell.
o Clinical findings
 Gynecomastia
 Spider telangiectasia
 Female distribution of hair
- Hepatic encephalopathy
o Reversible metabolic disorder
o Increase in aromatic amino acids (e.g., phenylalanine, tyrosine, tryptophan) which are then Converted into false neurotransmitters (e.g., γ-
aminobutyric acid)
o Increase in serum ammonia - Ammonia derives from metabolism of amino acids and from the release of ammonia by bacterial ureases in the bowel.
o Due to a defective urea cycle that cannot metabolize ammonia
o Precipitating factors:
o ↑protein intake, alkalosis, sedatives, portosystemic shunts
o Clinical findings
 (a) Alterations in the mental status
 (b) Somnolence and disordered sleep rhythms
 (c) Asterixis (i.e., inability to sustain posture, flapping tremor)
 (d) Coma and death in late stages

Alcoholic liver disease


- Accounts for 2/3rds of chronic liver disease in NA
- More than 10 million Americans are alcoholics
- Alcohol abuse causes 100,000 to 200,000 deaths annually in the US
- Fifth leading cause of death (20,000 due to end-stage cirrhosis) many due to automobile accidents
- 25% to 30% of hospitalized patients have problems related to alcohol abuse
Fatty change (Steatosis)
- Enlarged,yellow, greasy liver
- Reversible with alcohol abstinence.
- Increased NADH production.
- Triglyceride in hepatocytes pushes nucleus to periphery
- Centri-lobular steatosis (reversible) Eventual fibrosis around central vein (irreversible)
o Mechanism for alcohol induced fatty change in the liver :
 Increased glycerol 3 phosphate synthesis
 Increased acetyl CoA due to increased acetate Increased FA synthesis
 Decreased beta oxidation of FA
- A 60-year-old man has a 6-month history of abdominal swelling. On a daily basis, he smokes two packs of
cigarettes, drinks five cups of coffee, and reports that he consumes 2 sixpacks of beer. Physical examination
shows a distended abdomen with a palpable liver 2 cm below the costal margin. A liver biopsy is shown in the
image. If this patient becomes abstinent, his liver will most likely do which of the following? Revert to normal
Alcoholic hepatitis –
- Acute illness usually following a heavy drinking binge
- Pathogenesis
o Due to acetaldehyde damage to hepatocytes
o Stimulation of collagen synthesis around the central vein
 Perivenular fibrosis
- Clinically variable
o No symptoms
o RUQ pain, hepatomegaly, jaundice, malaise and anorexia
o Fulminant hepatic failure
- Microscopy
o Hepatocyte swelling (ballooning) and necrosis
o Mallory bodies-accumulation of intermediate
(cytokeratin) filaments
o Neutrophilic infiltration
o Fatty change & eventually Fibrosis around central vein
Alcoholic cirrhosis
- Develops in 15% of alcoholics
- Micronodular cirrhosis
- Big liver, small nodules
- Most common disease requiring liver transplantation in adults
Viral hepatitis
- Infection of liver caused by a group of viruses having affinity for
liver
- Most common infectious disease
- Asymptomatic, malaise and weakness, nausea, anorexia,
jaundice, and urine may be dark
- Symptomatic patients have 3 phases:
o Prodrome
o Icteric or nonicteric phase
o Recovery phase
- Lab findings:
o Markedly elevated ALT and AST; Usually ALT > AST
o Mixed hyperbilirubinemia, Specific diagnosis- Serology
- Gross- small liver, big nodules (macronodular)
Acute viral hepatitis
- Definition
o Signs and symptoms < 6 months
- Caused by any of the hepatitis viruses
- Gross findings:
o Enlarged reddened liver
o Greenish if cholestatic
- Microscopy
o Hepatocyte injury
 Ballooning degeneration (swelling)
 Hepatocyte necrosis
 Cytolysis (rupture) or apoptosis (shrinkage)
 Councilman bodies seen (arrows)
 Lobular disarray (normal architecture lost)/
 Hypertrophy/hyperplasia of Kupffer’s cells
 Portal tracts inflammation by Mononuclear cells
A previously healthy 38-year-old man complains of yellow discoloration of his eyes, abdominal pain, and low-grade fever of 1-month duration. Physical examination
demonstrates a distended abdomen, right upper quadrant tenderness, and a palpable liver edge 2 cm below the right costal margin. Total serum bilirubin is 7.4 mg/dL.
Serum levels of AST and ALT are elevated (229 and 495 U/L, respectively). The prothrombin time is prolonged (18 seconds). A liver biopsy is shown in the image. The arrows
point to Councilman bodies. The pathologic findings are indicative of which of the following liver diseases? Acute viral hepatitis

Chronic hepatitis
- Definition
o Signs and symptoms > 6 months
- Causes
o Hepatitis virus B, C and D
- Microscopy
o Chronic inflammation
o Spills into parenchyma → interface hepatitis (piecemeal necrosis and disruption of limiting plate)
o Bridging fibrosis
o Tendency to progress to cirrhosis
- Carrier state for HBV
o Liver architecture normal
o Isolated cells or clusters - “Ground-glass” hepatocytes (HBsAg in HBV)
Hepatitis viruses
- Hepatitis A virus (HAV)
o No chronic hepatitis /carrier state
o Rarely causes fulminant hepatitis
o Fatality rate -0.1%.
o HAV viremia is transient, so blood-borne transmission of HAV occurs only rarely
o Donated blood is not screened for HAV
o Hepatitis A serology
 Acute or recent infection- Anti-HAV IgM
 Prior infection or immunization- Anti-HAV IgG
- Hepatitis B virus (HBV)
o Acute hepatitis with resolution
o Chronic hepatitis →cirrhosis
o Fulminant hepatitis with massive liver necrosis
o Backdrop for hepatitis D virus infection
- A 30-year-old man presents with a 9-month history of fatigue and recurrent fever. He also complains of yellow skin and sclerae, abdominal tenderness, and dark
urine. Physical examination reveals jaundice and mild hepatomegaly. Laboratory studies demonstrate elevated serum bilirubin (3.1 mg/ dL), decreased serum
albumin (2.5 g/dL), and prolonged prothrombin time (17 seconds). Serologic tests reveal antibodies to hepatitis B core antigen (IgG anti-HBcAg). The serum is
positive for HBsAg and HbeAg. A liver biopsy is shown in the image. What is the most likely diagnosis? Chronic hepatitis

Disorders of the Liver & Biliary Tract-5


Primary biliary cirrhosis
- Definition
o Chronic liver disease of unknown etiology (autoimmune)
o Lympho-plasmacytic inflammation of portal tracts and granulomatous destruction of intra-hepatic bile ducts
- Epidemiology
o Males : females= 1:10, age 30-65 yrs
- Presentation
o Middle-aged women
o Obstructive jaundice
o Pruritis
o Xanthomas, xanthelesmas, and elevated serum cholesterol
o Fatigue
o Cirrhosis (late complication)
o Do have other autoimmune diseases
 RA, scleroderma, or SLE
- Gross: Yellow green, fine, granular
- Laboratory findings
o Elevated conjugated bilirubin
o Transaminasemia, elevated alkaline phosphatase and 5’-nucleotidase
o Elevated IgM antibodies
o Anti-mitochondrial antibodies (AMA) >90%
Secondary biliary cirrhosis
- Etiology
o Extra-hepatic bile duct obstruction (gallstones)
o Biliary atresia
o Strictures
o Carcinoma of pancreatic head
- Sex predilection
o None
- Clinical features
o Pruritus, jaundice, malaise, dark urine, light stools, hepatosplenomegaly
- Pathology
o Gross liver (end-stage)
 Yellow-green
 Cirrhosis
o Microscopy
 Prominent bile stasis in bile ducts
 Bile duct neutrophilic infiltration-- abscess
 Peri-portal fibrosis --- cirrhosis
o Lab findings
 Conjugated hyperbilirubinemia
 Increased serum alkaline phosphatase
 Increased bile acids and cholesterol

Wilson disease (hepato-lenticular degeneration)


- Definition
o Genetic disorder of copper metabolism
 Accumulation of toxic levels of copper in various organs
- Genetics
o Autosomal recessive (chromosome 13)
o WD gene (ATP7B)
 codes for a hepatocyte canalicular copper-transporting ATPase
- Pathogenesis
o Gene mutation
 Defective hepatocyte transport of Cu into bile for excretion
 ↓ed synthesis of ceruloplasmin (binding protein for Cu in blood)
o Unbound Cu accumulates in blood
 Loosely attached to albumin
 Cu deposits in tissues --- toxic effect
o Clinical presentation
 In childhood or adolescence with liver disease
o Cornea
 Kayser-Fleischer rings (Cu deposition in Descemet’s membrane)
- A 15-year-old boy complains of a 2-month history of fatigue, abdominal pain, and yellow eyes and skin. Physical examination shows tremor of his hands, lack of
coordination, and mild jaundice. The results of an ophthalmic examination are shown in the image. This patient most likely has an inborn error of
metabolism associated with tissue overload of which of the following elements? Copper
- Distribution of disease
o Liver: Chronic hepatitis, Micro-nodular cirrhosis
o Brain
 Cu deposits in putamen
 Movement disorder resembling parkinsonism
 Cu deposits in sub-thalamic nucleus
 Hemi-ballismus
 Cu is toxic to neurons in cerebral cortex
 Produces dementia
o Diagnosis
 ↓ed total serum Cu
 Due to decreased ceruloplasmin
 ↓ed serum ceruloplasmin
 Useful in diag. in early stages
 ↑ed serum/urine free Cu
 Useful in diag. in later stages
o Treatment
 Copper-chelators
 Liver transplant is curative
Hemochromatosis
- Definition
o Increased levels of iron → tissue injury
- Hereditary (primary)
o AR (HLA-H gene on chr 6p)
o Mechanism
 Increased iron absorption
- Secondary
o Eg. Transfusions for chronic anemias
- Distribution of disease
o Liver
 Micro-nodular cirrhosis
 HCC (risk-200 times greater)
o Pancreas
 Diabetes mellitus
o Skin
 Hyper-pigmentation
o Heart
 Congestive heart failure
 Cardiac arrhythmias
o Gonads
 Hypo-gonadism
- Diagnosis
o Markedly elevated serum iron and ferritin;
o Prussian blue stain
 Increased tissue iron levels (liver biopsy)
- Treatment
o Phlebotomy
- Increased transferrin saturation-Decreased TIBC
Alpha-1 antitrypsin deficiency
- Definition
o Autosomal recessive – Low serum levels of α-1-AT
o Production of a mutant protein that cannot be secreted into blood
o α-1-AT accumulates in hepatocytes → liver damage
- Clinical findings in children with PiZZ variant
o Neonatal hepatitis with intra-hepatic cholestasis
o Most common cause of cirrhosis in children
o Increased risk for hepatocellular carcinoma
- Genetics: α-1-AT
o Produced by Pi gene (chr 14)
 PiM: normal, most common form (90%)
 PiS deficiency: mildly reduced levels
 PiZ deficiency: markedly reduced levels
o Homozygous PiZZ
 Severe reductions (15% of normal) in enzyme levels
- Distribution of disease
o Liver
 Micro-nodular cirrhosis
 Increased risk of HCC
 Treatment- Liver transplant
o Lungs
 Pan-acinar emphysema
o Figure: PAS positive, eosinophilic cytoplasmic globules within hepatocytes
Autoimmune hepatitis
- Anti-smooth-muscle antibodies and antinuclear
- antibodies positive.
- LE cell test may be positive.
- All serologic tests for viral markers are within normal limits.
- No granulomas; no evidence of fibrosis.
- Elevated liver enzymes (AST and ALT).
Neonatal hepatitis
- Epidemiology
o Idiopathic
o Associated with infections (e.g., CMV), biliary atresia & inborn errors of metabolism (e.g., α1-AT def- 30 % of
cases)
- Biopsy shows multinucleated giant cells
o "Giant cell" hepatitis
Reye syndrome
- In children < 4yrs following acute febrile viral infection (varicella, influenza)
- Almost always associated with aspirin intake
- Liver- Micro-vesicular fatty change without nuclear displacement
- Brain- Encephalopathy, edema, coma, convulsions
- Rare, potentially fatal disease
- Mechanism
o Unknown
o Mitochondrial injury and dysfunction
o Disruption of urea cycle
 Increase in serum ammonia
o Defective β-oxidation of fatty acids
- Lab findings:
o Transaminasemia,
o Normal /slight increased in total bilirubin, Increased serum ammonia
- Complete recovery 75%
- May lead to permenant neurological deficits, coma
- Treatment supportive
Non-alcoholic fatty liver disease (NAFL)/Non-alcoholic steatohepatitis (NASH)
- Fatty change (macrovesicular steatosis) in absence of heavy alcohol consumption
- Steatosis + inflammation (neutrophils)
- Associated conditions:
o Obesity, Dyslipidemia, Diabetes mellitus type II.
o Abdominal CT scan-decreased attenuation of the entire enlarged liver due to fat deposition
- Clinical course
o Asymptomatic
o Persistent elevation of liver enzymes
o 10-30% patients develop cirrhosis
 Most common cause of "cryptogenic“ cirrhosis
o May progress to hepatitis C viral infection
- Steatosis Steato-hepatitis Cirrhosis

Budd Chiari Syndrome (hepatic vein thrombosis)


- Definition
o Occlusion of hepatic vein by a thrombus → death
- Etiology
o Thrombotic conditions
 Polycythemia vera
 Pregnancy
 Oral contraceptives
 PNH
 HCC invades hepatic vein
 Idiopathic
 Bone marrow transplantation
- Clinical
o Abdominal pain, hepatomegaly, ascites, and death
- Gross
o Thrombosis of hepatic veins
o Extreme retention of blood in the liver
- Microscopy
o Centri-lobular congestion
- Pathology
o Gross
 Nutmeg pattern
 Alternating dark (congested central areas) and light (portal tract areas)
 Painful hepatomegaly with or without jaundice
 Increased transaminases
o Microscopy
 Congestion of central veins and sinusoids
 Necrosis of hepatocytes around central vein
- Complications
o Centrilobular necrosis
 Ischemic necrosis of centrilobular hepatocytes
o Long-standing congestion
 ----Centrilobular fibrosis---- cirrhosis
Peliosis hepatis
- Sinusoidal dilation-randomly distributed blood filled cavities through out the liver
- Associated with exposure to
o Anabolic steroids, OCP, Danazol
o Bartonella henselae causing bacillary angiomatosis
 Occurs in AIDS
- Pathology
o Liver- mottled, blotchy with irregular blood-filled lakes
o M/S- Irregular blood-filled cystic spaces
- Potential for intra-peritoneal hemorrhage
Polycystic Liver Disease
- Multiple diffuse cystic lesions
o Lining- Cuboidal or flattened biliary epithelium
o Content- Straw-colored fluid
- Associated with PCKD
Congenital Hepatic Fibrosis
- Liver
o Dense fibrous septa with embedded irregular biliary structures
- Origin
o Incomplete involution of embryonic ductal structures
Hepatic adenoma (Liver cell adenoma)
- Young women
- Related to oral contraceptive use
- Clinical Findings:
o Sub-capsular adenomas may rupture
o →intra-peritoneal hemorrhage
- May regress after oral contraceptives are discontinued
Hepatoblastoma
- Most common liver tumor of young childhood
- >80%- β-catenin mutation
- Malignant
- Epithelial features
o Of fetal liver
- Mixed features
o Of epithelial and mesenchymal differentiation
Hepato-cellular carcinoma (HCC
- Most common primary tumor of liver of adults
- Asia and Japan > United States
- Etiology
o Cirrhosis, HBV, HCV, chronic alcoholism, aflatoxin B1, aromatic amines and azo Dyes.
- C/F
o Hepatomegaly
o RUQ pain
o Weight loss
- Clinical course
o Death within 6 months of diagnosis
- Tumor marker
o α- fetoprotein (AFP)
- Fibro-lamellar variant
o Younger age
o Fibrous bands
o Better prognosis
Cholangiocarcinoma
- Risk factors
o Thorotrast
o Clonorchis sinensis (liver fluke)
o Caroli disease
o Primary sclerosing cholangitis
- Pathology
o Gross- Pale
o M/S - Adenocarcinoma arising from bile duct epithelium
- Clinical course
o Discovered late during the course
o Poor prognosis (average survival is 6 months)
Angiosarcoma
- Rare, malignant, vascular neoplasm
- Chemical carcinogens
o Vinyl chloride, thorotrast, arsenic
- Aggressive tumors with a poor prognosis
Metastatic tumors to liver
- Most common tumor found within liver in adults
- Common primary sites
o Colon, breast and lung
o Melanoma
- Gross
o Multiple well circumscribed masses
o Important GF for enlargement of metastatic lesions – VEGF
- A 69-year-old woman arrives in the emergency room complaining of weakness, abdominal pain, and a 9
kg (20 lb) weight loss during the past month. Physical examination reveals jaundice, conspicuous
hepatomegaly, and ascites. The patient expires, and a section of liver is examined at autopsy (shown in the image). Which of the following is the most likely
diagnosis? Metastatic carcinoma of the liver
Schistosomiasis
- Schistosoma mansoni
- Africa and Latin America
- The larvae enter the superior mesenteric vein and from there into the portal vein. In the portal vein, they develop into adult worms that deposit eggs (have a
sharp lateral spine) to which the host develops an inflammatory response marked by concentric fibrosis (“pipestem cirrhosis”) in the vessel wall.
- Complications of cirrhosis include portal hypertension, ascites, and esophageal varices.
- Because it is an invasive helminthic infection, eosinophilia is present.
- Treatment is praziquantel.
Amebic liver abscess
- Rare in the US except for recent immigrants from Mexico, South America, India, etc.
- Organism
o Entamoeba histolytica
- Gross
o Necrotic abscess filled with brown paste-like material (“anchovy sauce”)
- Treatment
o Antibiotics ± surgical drainage
Hydatid cyst- liver

Pathology of Gallbladder
Congenital Anomalies
- Congenital absence
- “Phrygian cap”
o A folded fundus
o Most common anomaly
- “Biliary atresia"
o Hypoplastic narrowing of biliary channels
Biliary atreasia
- Complete obstruction of lumen of extra-hepatic biliary tree within 1st 3 months of life
- Progressive destruction of the bile ducts leading to scarcity of bile ducts
- Cause of neonatal cholestasis- occurs in 1:10,000 live births
- Single most frequent cause of death from liver disease in early childhood
Cholelithiasis (gallstones)
- >80% are "silent"
o No biliary pain or stone complication
- Types of gallstones:
o Cholesterol stones
 In the West (80%)
 > 50% of crystalline cholesterol monohydrate
o Pigment stones
 Bilirubin calcium salts
Gall stones
- Right upper quadrant pain with nausea and vomiting.
- Abdominal CT scan reveals a polypoid
- mass of the gallbladder protruding into the lumen, diffuse thickening of the gallbladder wall, and enlarged lymph nodes.
- Risk factors:
o Cholesterol stones
 Female over 40 years old
 Obesity, use of lipid-lowering drugs (clofibrate)
 Cholesterol excretion is increased in bile
 Native Americans (e.g., Pima and Navajo Indians)
o Pigment stones
 Black pigment stones
 S/O chronic extra-vascular hemolytic anemia (sickle cell anemia)
 Excess bilirubin in bile produces Ca bilirubinate
 Brown pigment stones
 Sign of infection in CBD

- Pathogenesis of Cholesterol Stones
o Super-saturation of bile with cholesterol
o GB hypomotility promotes nucleation into solid cholesterol monohydrate crystals
o Mucus hyper-secretion in GB-traps crystals
o Aggregation into stones within gall-bladder mucous layer
- Pathogenesis of Pigment Stones
o Composition
 Complex mixtures of insoluble Ca salts of unconj. bilirubin along with inorganic Ca salts
o Unconjugated bilirubin
 Minor component of bile but ↑es in infections of biliary tract. E. coli, Asc. lumbricoides , C.sinensis
 Release of microbial β-glucuronidases, which hydrolyze bilirubin glucuronides
o Intravascular hemolysis
 ↑ed hepatic secretion of conjugated bilirubin
- Cholesterol stones
o Arise exclusively in GB
 Composed of 100% pure cholesterol (radiolucent) & few having calcium salts (radio-opaque)
o Pure cholesterol stones
 Pale yellow, round to ovoid
 finely granular, hard external surface
 C/S- glistening radiating; crystalline
o With ↑ing proportions of Ca carbonate, phosphates, and bilirubin
 Discoloration
 C/S- Lamellated, gray-white to black
- Pigment gallstones
o Black pigment stones:
 In sterile GB bile
 < 1.5 cm diameter, multiple
 Spiculated, molded
 Oxidized polymers of Ca salts of unconj. bil.
 Lesser amts of CaCO3, CaPO4, mucin glycoprotein
 Radio-opaque( due to Ca CO3 and CaPO4)
o Brown stones
 In infected extra-hepatic or intra-hepatic ducts
 Laminated
 Consistency- Soft; soap-like or greasy
 Composition
 Pure Ca salts of unconjugated bilirubin, mucin glycoproteins, cholesterol fraction, &
Ca salts, therefore radiolucent
Strawberry gallbladder/Cholesterolosis
- Cholesterol hypersecretion by liver
- Promotes excessive accumulation of cholesterol esters within lamina propria of GB
- Mucosa
o Studded with minute yellow flecks containing many foamy macrophages →“strawberry gallbladder”
o 70- 80% patients- asymptomatic throughout life
o Pain in right hypochondrial region
 excruciating and constant or colicky (spasmodic)
- Abdominal Pain
o Mr Gut Feeling, a 45-year old male presents with severe upper abdominal pain radiating to back, after an alcoholic binge
o Mrs Feeling, a 40-year old fat, flatulent female presents with severe pain in right upper abdomen, and vomiting
o Junior Feeling, a 15-year old boy presents with pain abdomen, which started around umbilicus and shifted to right lower abdomen, associated with
nausea, vomiting, and fever
- Neotrophilic leukocytosis with left shift
o Gross features of gallbladder showed enlarged and tense organ and bright red discoloration due to subserosal hemorrhages and serosa covered by
fibrin.
o Microscopy showed neutrophils infiltrating mucosa and submucosa of gallbladder.
Cholecystitis
- Inflammation of GB
o Acute
o Chronic
o Acute superimposed on chronic
- Usually associated with gallstones- cystic duct (90%); Mucosal ulceration predisposes to infection (usually E. coli).
- Other causes- AIDS with CMV or Cryptosporidium & severe volume depletion
Morphology- Acute cholecystitis
o No difference b/w acute acalculous and calculous cholecystitis
o Gross – Gall bladder
 Enlarged and tense
 Bright red or green-black discoloration due to subserosal hemorrhages
 Serosa covered by fibrin
o Neutrophils infiltrate mucosa and submucosa of gallbladder
o Mild cases- wall is thickened, edematous, and hyperemic; severe cases- gangrenous; Empyema-
pus.
- Clinical features
o Fever with nausea and vomiting
 Usually 15 to 30 minutes after eating
o Pain
 Initial mid-epigastric colicky pain
 Pain eventually shifts to RUQ
 Pain is constant and dull - may radiate to right scapula
 Attack usually subsides in 7-10 days and freq. within 24 hours
o Jaundice
 Suggests a stone in CBD
- Laboratory findings
o Neutrophilic leukocytosis with left shift
o Elevation in serum alkaline phosphatase
o Tests to identify stones
 USG - Gold standard
 Radionuclide scan identifies stone(s) in cystic duct
- C/F - Acute acalculous cholecystitis
o Symptoms obscured
 by underlying conditions precipitating the attacks
o Incidence of gangrene/ perforation much higher
 than in calculous cholecystitis
o Acute emphysematous cholecystitis
 Invasion of gas forming organisms, clostridia / coliforms
Chronic Cholecystitis
- Sequel to repeated attacks of mild to severe acute cholecystitis
- Super-saturation of bile predisposes to
o chronic inflammation and stone formation
- Chemical inflammation / micro-organisms - E. coli, enterococci
- Clinical fearures
o Recurrent attacks of either steady or colicky epigastric or RUQ pain, with nausea, vomiting and intolerance for food, esp. fatty diet.
o Complications:
 Bacterial super-infection with cholangitis or sepsis
 Gallbladder perforation and local abscess
 Gallbladder rupture with diffuse peritonitis
 Cholecystoenteric Fistulas
- Porcelain gallbladder
o Extensive dystrophic calcification in GB wall with increased incidence of associated cancer
- Hydrops of GB
o Atrophic, chronically obstructed GB containing clear secretions (mucus), with occasional
cholesterol stones
- Figure: Xanthogranulomatous cholecystitis shrunken, nodular, chronically inflammed gallbladder
with foci of necrosis and hemorrhage
Choledocholithiasis
- Presence of stones within bile ducts of biliary tree
- Epidemiology
o Asia - Higher incidence of primary stone formation in biliary tree - pigmented (due to biliary
tract infections)
- Symptoms
o Obstruction, pancreatitis, cholangitis, hepatic abscess, secondary biliary cirrhosis, acute calculous cholecystitis
Cholangitis
- Bacterial infection of bile ducts
- Due to any lesion that creates obstruction to bile flow, most commonly choledocholithiasis
- Bacteria- Enteric Gram-negative aerobes- E. coli, Klebsiella, Clostridium, Bacteroides, or Enterobacter and Group D streptococci
- Clinical findings:
o Fever, chills, abdominal pain, and jaundice
Choledochal cysts
- Congenital dilations of CBD
o Cysts- segmental or cylindrical dilation
- Clinical findings:
o In children < 10 yrs
o Nonspecific symptoms - jaundice and/or recurrent abdominal pain that are typical of biliary colic
- Complications
o Predispose to stone formation, stenosis and stricture, pancreatitis, obstructive biliary complications within liver
Caroli disease
- Autosomal recessive disease
- Segmental dilatation of intra-hepatic bile ducts
- Clinical findings
o Asso. with polycystic kidney disease
o Increased incidence of cholangiocarcinoma
Carcinoma of gallbladder
- Epidemiology
o Dominant in elderly women (in 7th decade)
o Preop. diagnosis is rare, occurring in < 20% of patients
o Insidious onset; ~ cholelithiasis
o Usually adenocarcinomas- 2 patterns of growth- infiltrative & exophytic
o Invade liver, cystic duct, bile ducts and porta-hepatic LN & seedings in peritoneum, GIT, lungs
o Poor prognosis
- Pathogenesis
o Cholelithiasis (95% of cases)
 Due to irritative trauma and chronic inflammation
 Carcinogenic derivatives of bile acids also may play a role (cholic acid, deoxycholic acid, taurocholic acid)
o Porcelain gallbladder
 Gallbladder with dystrophic calcification

Pathology of pancreas
Pancrease
- Fleshy retroperitoneal organ.
- Exocrine part
o secretes digestive enzymes.
- Endocrine part
o Islets of Langerhans– secretes diff. hormones- glucagon & insulin.
- Thin capsule that sends septa penetrating the gland---- lobules.
Exocrine pancreas
- Compound acinar gland
o Secretory acini
o Duct system
- Acini
o Radial orientation of pyramidal exocrine acinar cells with basophilic cytoplasm
- Apical portion
o contains ‘zymogen’ granules filled with precursors of digestive enzymes
Acute pancreatitis
- Group of reversible lesions
- Inflammation of pancreas
o Edema and fat necrosis with hemorrhage
- 80% cases in Western countries associated with
o Biliary tract disease
o Alcoholism
- Male-female ratio
o 1:3 in pts with biliary tract disease
o 6:1 in pts with alcoholism
- Pathogenesis:
o Auto-digestion of pancreatic substance by inappropriately activated pancreatic enzymes
o Brush border enzyme, enteropeptidase (enterokinase)
 cleaves trypsinogen (Important triggering event) → trypsin (active form)
o Activated trypsin
 Catalyzes /activates proenzymes (inactive enzymes) → active enzymes that leads to autodigestion of pancreas by disintegration of fat cells,
vessels inflammation, thromboses & rupture
 Amylase and lipase do not require trypsin activation as they are secreted in an active form
- Gross Pathology- Pancreas
o Firm, edematous
o Foci of hemorrhage and yellow white fat necrosis
o Completely necrotic → soft, flabby and hemorrhagic due to digestion of vessel walls by pancreatic
enzymes
o Chalky white surface -saponification- chelation of Ca with fatty acids liberated by pancreatic enzymes

- Activated enzymes
o Trypsin - important in activation of pro-enzymes.
o Proteases -- damage acinar cell structure
o Lipases and phospholipases --- enzymatic fat necrosis.
o Elastases --- damage vessel walls – h’ge
o Activated enzymes also circulate in blood
- Clinical features
o Fever, nausea, vomiting
o Severe, boring pain with radiation to back
o Grey- Turner’s sign (flank hemorrhage)
o Cullen’s sign (peri-umblical hemorrhage)
o Most patients recover fully; 5% die of shock in 1st week
o Complications:
 Pancreatic pseudocyst- Collection of digested necrotic-hemorrhagic tissue around
pancreas with an abdominal mass with persistence of serum amylase >10 days;
lack an epithelial lining ("pseudo") & lined by fibrosed granulation tissue
 Pancreatic abscess
 Atelectasis, ARDS
 Tetany
 Shock
 DIC
- Laboratory findings
o Marked elevation of serum amylase levels
 In 2-12 hrs
 N in 2-3 days
 In urine for 1-14 days
o Rising serum lipase level
 ↑ed in 3-4 days
 More specific
 N in 5-6 days
 Not excreted in urine
o Glycosuria
 Hyper-glycemia (destruction of β-islet cells)
 10% cases
o Hypocalcemia
 Ca soaps precipitate (fat necrosis)
Chronic Pancreatitis
- Repeated attacks of moderately severe abdominal or back pain in middle-aged males with inflammation of pancreas & destruction of exocrine parenchyma, and
fibrosis & in late stages- destruction of endocrine parenchyma leading to irreversible impairment in pancreatic function
- Mostly idiopathic
- Alcohol abuse-most common known cause
- Cystic fibrosis-most common cause in children
- Malnutrition-most common cause in developing nations
- Increased amylase and lipase
- Pancreatic calcifications (CT scan best study)
Hereditary pancreatitis
- Recurrent attacks of severe pancreatitis usually beginning in childhood
- Germ line (inherited) mutations in
o Cationic trypsinogen gene (PRSS1)
o Serine Protease Inhibitor, Kazal Type 1 (SPINK1)
Congenital cysts
- Due to anomalous development of pancreatic ducts
- Associated with
o Congenital polycystic disease
o von Hippel-Lindau disease
 Vascular neoplasms in retina, cerebellum, brain stem
 Pancreatic cysts
- Pathology
o 3-5 cm in diameter lined by cuboidal epithelium; thin, fibrous capsule
o Contain a clear-turbid mucoid / serous fluid
- VHL syndrome
o Deletion of chr 3p
o Hemangioblastomas of cerebellum and retina
o Cysts
o Bilateral RCC (clear cell type)
Pancreas cystic neoplasms
- 5 - 15% of all pancreatic cysts are neoplastic (most cysts are pseudocysts)
- Serous cystadenoma
o Benign
- Mucinous cystic neoplasms
o Benign, borderline malignant, or malignant
Serous cystadenoma
- Benign cystic neoplasms
o Glycogen-rich low-cuboidal cells surrounding small cysts with clear/thin/straw-colored fluid
- 25% of all cystic neoplasms of pancreas
- C/F
o Abdominal pain
o Palpable abdominal masses
Mucinous cystic neoplasms
- Almost always arise in women
- Benign, borderline malignant, or malignant
- Location
o Body or tail of pancreas
- C/F
o Painless, slow-growing masses
- M/S of cysts
o Lumen - thick, tenacious mucin
o Lining- columnar mucinous epithelium
Intra-ductal Papillary Mucinous Neoplasms (IPMNs)
- Involve larger pancreatic ducts
- Produce cysts containing mucin
- Benign, borderline malignant, or malignant
- Location
o Head of pancreas
o Mucin oozes out
Precursors to Pancreatic Cancer
- Non-neoplastic epithelium → histologically well-defined non-invasive lesions in small ducts and ductules “Pancreatic intra-epithelial neoplasias" (PanINs)
Invasive carcinoma
- Epidemiology
o ↑ing in frequency in Western countries
o Incidence
 1 per 10,000 of population in most Western countries
o 4th most common cause of cancer deaths in U.S.A
o Age/Sex
 50+
 M>F
- Etiology
o Smoking
 Doubles the risk of pancreatic cancer
o Diet
 Rich in fats
o Chronic pancreatitis and diabetes mellitus
 Associated with increased risk
- Pathology
o Location
 Head (80%), body (15%), tail (5%)
o Gross
 Hard, gray-white, poorly defined masses
o Microscopy
 Ductal adenoCa
o Characteristic features
 Highly invasive
 “Desmoplastic response”
o Hard, gray-white, poorly defined mass ( arrow)
o Poorly formed glands in a densely fibrotic stroma and few inflammatory cells
- Clinical and laboratory findings
o Epigastric pain with weight loss
 Signs of CBD obstruction (carcinoma of head of pancreas)
 Jaundice (CB > 50%)
 Light-colored stools (absent UBG)
 Palpable, enlarged, nontender GB (Courvoisier's sign)
o Superficial migratory thrombophlebitis
 Increased CA19-9
 Gold standard tumor marker
Metastasis
- Lymph nodes
o frequently involved
- Liver
o often enlarged due to metastasis
- Distant metastasis
o Lungs and bones
- Diagnosis
o Imaging techniques
 Endoscopic USG, CT
o Per-cutaneous needle biopsy
o K-RAS oncogene
 Mutated in 90% pancreatic cancers
o Serum levels
 ** CEA and CA19-9 are elevated, but are neither specific nor sensitive
- A 60-year-old woman complains of increasing abdominal girth of 4 weeks in duration. Physical examination discloses
ascites, and cytologic examination of the fluid reveals malignant cells. Exploratory laparotomy shows multiple tumor
nodules on the serosal surface of the intestines. Which of the following is the most likely diagnosis? Metastatic
carcinoma
Pancreatoblastoma
- Rare malignant neoplasms
- In children
- Both acinar and squamoid nests
- Fetal structures

You might also like